You are on page 1of 37

Crazy Complete Compendium of AIIMS May 2010

© Dr Saraswata Das. All Rights Reserved.

Preface

Dear Friends,
Crazy Hope u all enjoyed writing the
AIIMS May 2010 Paper! It was
Complete really an exhilarating experience
for all of us. This paper can
Compendium undoubtedly be termed as the
“craziest” of the AIIMS Paper till
of now…so many repeats were there
that we had to literally hunt for
AIIMS May the new questions! But then
again, those Profs at AIIMS know
2010 very well how to fool the students.
Some questions had their
“EXCEPT” omitted, and whether
they did it intentionally or it was
the printer’s devil…we can at best
speculate. Some questions though
looked easy, had some twists in
them. And last but not the least, to
add to the confusion, some repeat
questions were there whose
answers as given in the Great
Grandfathers’ books (AA & MK)
are still controversial. To sum up,
it was really a crazy paper which
only AIIMS can set!

So here’s the crazy solution to all


those crazy questions. I am
grateful to all my friends of
CNMC Medicos02 & those at
various forums who helped me to
recollect the questions &
encouraged me to write this book.
Dr Saraswata Das And if u like (or don’t like!) this
MD Radiodiagnosis (Std)
E-book, feel free to drop a line at
saraswatastar@yahoo.com .
1
Online support at: www.saraswatastar.weebly.com
Crazy Complete Compendium of AIIMS May 2010
© Dr Saraswata Das. All Rights Reserved.

Crazy Complete Compendium of AIIMS May 2010


Dr. Saraswata Das
MD Radiodiagnosis (Std)

ANATOMY

1. Middle superior alveolar nerve is a c) Anteromedial to lumbar


branch of sympathetic chain
a) Mandibular division of d) Posterolateral to lumbar
trigeminal nerve sympathetic chain
b) Palatine division of maxillary
nerve 8. Paneth cells – True is
c) Anterior nasal division of a) Rich in rough endoplasmic
maxillary nerve reticulum
d) Inferior alveolar nerve b) High zinc content
c) Foamy cytoplasm
2. All the following muscles retracts the d) Numerous lysozyme granules
scapula EXCEPT
a) Trapezius 9. About sternocleidomastoid tumor all are
b) Rhomboid major true except –
c) Rhomboid minor a) Always associated with breech
d) Levator scapulae b) Spontaenous resolution in
most cases
3. Cranial nerve NOT carrying c) Two-third have palpable neck
parasympathetic fibres mass at birth
a) 4th d) Uncorrected cases develops
b) 7th plagiocephaly
c) 3rd
d) 9th 10. The function of 8th cranial nerve is
related to
4. Prostatic urethra – True are A/E a) Smell
a) Trapezoid in cross section b) Taste
b) Elevated round swelling called c) Touch
verumontanum d) Balance
c) Opening of prostatic ducts
d) Posterior part has urethral 11. Anterior ethmoidal nerve supplies all
crest except?
a. Maxillary sinus
5. Morgagni hernia presents most a) Interior of nasal cavity
commonly on b) Dural sheath of anterior cranial
a) Left posterior fossa
b) Right anterior c) Ethmoidal air cells
c) Right posterior
d) Left anterior 12. A healthy young athlete is sitting at the
edge of the table with knee at 90 degree
6. Meralgia parasthetica is due to flexion. He fully extends it. What will
involvement of happen?
a) Sural nerve a) Movement of tibial tuberosity
b) Medial cutaneous nerve of towards lateral border of
thigh patella
c) Lateral cutaneous nerve of b) Movement of tibial tuberosity
thigh towards medial border of
d) Peroneal nerve patella
c) Movement of tibial tuberosity
7. Celiac plexus is located towards centre of patella
a) Anterolateral & around the d) No change in position
aorta
b) Posterolateral & around the 13. Pain insensitive structure in brain is
aorta a) Falx cerebri

2
Online support at: www.saraswatastar.weebly.com
Crazy Complete Compendium of AIIMS May 2010
© Dr Saraswata Das. All Rights Reserved.

b) Dural sheath surrounding


vascular sinuses 22. Spinocerebellar tract – Function is
c) Choroid plexus a) Equilibrium
d) Middle meningeal artery b) Vision
c) Initiation & planning of
14. Appendices epiploiceae present in movements
a) Appendix d) Smoothens & co-ordinates mvt
b) Cecum
c) Rectum 23. Chemotaxis – Main feature is
d) Sigmoid colon a) Unidirectional locomotion of
neutrophils
15. Pelvic splanchnic nerve supplies A/E b) Increase in phagocytosis
a) Appendix c) Increased random motion of
b) Rectum neutrophils
c) Uterus d) Increased adhesiveness to
d) Urinary bladder intima

PHYSIOLOGY
BIOCHEMISTRY
16. Appetite is stimulated by A/E
a) Agouti related peptide 24. Splicing activity seen in
b) Melanocyte concentrating a) mRNA
hormone b) snRNA
c) Melanocyte stimulating c) tRNA
hormone d) rRNA
d) Neuropeptide Y
25. After digestion by restriction
17. Capacitation occurs in endonucleases DNA strands can be
a) Uterus joined again by
b) Seminal vesicle a) DNA polymerase
c) Epididymis b) DNA ligase
d) Vas deferens c) DNA topoisomerase
d) DNA gyrase
18. The main cause of increased blood flow
to exercising muscles 26. Acetyl coA can be directly converted to
a) Raised blood pressure all except
b) Vasodilatation due to local a) Glucose
metabolites b) Fatty acids
c) Increased sympathetic c) Cholesterol
discharge to peripheral vessels d) Ketone bodies
d) Increased heart rate
27. After overnite fasing,levels of glucose
19. Intrinsic factor of Castle is secreted by tranporters reduced in
a) Chief cells a) Brain
b) Parietal cells b) Muscle
c) Mucus cells c) Adipocytes
d) B cells d) Hepatocyte

20. Somatomedin mediates 28. Insulin stimulates A/E


a) Deposition of chondroitin a) Glycolysis
sulphate b) Glycogenesis
b) Lipolysis c) Lipogenesis
c) Gluconeogenesis d) Ketogenesis
d) Decreased rate of glucose
uptake by cells 29. Eukaryotic cell membrane contains A/E
a) Cholesterol
21. Hyperaldosteronism associated with all b) Triglycerides
except c) Carbohydrates
a) Hypernatremia d) Lecithin
b) Hypokalemia
c) Hypertension 30. All of the following r true bout sickle cell
d) Metabolic acidosis trait except

3
Online support at: www.saraswatastar.weebly.com
Crazy Complete Compendium of AIIMS May 2010
© Dr Saraswata Das. All Rights Reserved.

a) Single nucleotide change leads 37. Damage produced by a bullet is related


to a change from glutamate to to its
valine a) Mass
b) A single base pair change b) Velocity
leads to RFLP c) Shape
c) Deoxygenated Hb leads to the d) Size
exposure of sticky end due to
replacement of non polar 38. A child brought with suspected ingestion
residue by polar residue presenting with dry mouth, dilated pupil ,
d) Offers protection to malaria in difficulty in swallowing, delirium, dry &
heterozygotes warm skin. The substance is
a) Anti-cholinergic
31. Hepatomegaly is the essential feature b) Sympathetic
for all of the following except c) Cholinergic
a) Hepatic porphyria d) Alpha blocker
b) Niemann Pick disease
c) Von Gierke’s disease
d) Hurler syndrome SPM

32. A 40 yr old woman presents with 39. Global warming true is


progressive palmoplantar pigmentation. a) CO2 is the major greenhouse
X-ray spine shows calcification of gas
intervertebral discs. On adding b) Stratosphere ozone layer is
Benedict’s reagent to urine, it gives harmful
greenish brown precipitate and blue c) CFC increases stratosphere
black supernatant fluid. What is the ozone layer
diagnosis? d) Kyoto protocol called for
a) Phenylketonuria 20%reduction in greenhouse
b) Alkaptonuria effect
c) Tyrosinemia type 2
d) Argininosuccinic aciduria 40. Vaccine with maximum efficacy
a) OPV
b) Measles
FORENSIC c) BCG
d) TT
33. Incised wounds on genitals are usually
a) Homicidal 41. NOT seen in fecal pollution
b) Self-inflicted a) Staph
c) Post-mortem artefact b) Strepto
d) Accidental c) E.coli
d) Clostridium perfringens
34. Heat rupture characterised by
a) Irregular margin 42. True about cow's milk are all except
b) Clotted blood a) Cow's milk contains 80% whey
c) Regular margin protein and not casein
d) Clotted blood vessels b) Has more protein than breast
milk
35. Motorcyclist fracture – True is c) Has more K+ and Na+ than
a) Ring fracture infant
b) Sutural separation d) Has less carbohydrates than
c) Comminuted fracture of vault of mothers milk
skull
d) Fracture base of skull into 43. Which of the following about lepromin
anterior & posterior halves test is not true?
a) It is negative in most children in
36. Commonest organ injured in blast injury first six months
a) Lung b) It is a diagnostic test
b) Liver c) It is an important aid to classify
c) Spleen type of leprosy disease
d) Pancreas d) BCG vaccination may convert
lepra reaction from negative to
positive

4
Online support at: www.saraswatastar.weebly.com
Crazy Complete Compendium of AIIMS May 2010
© Dr Saraswata Das. All Rights Reserved.

52. Vitamin A prophylaxis to a child is


44. Not spread by louse a) Health promotion
a) Epidemic typhus b) Specific protection
b) Q fever c) Primordial prevention
c) Relapsing fever d) Secondary prevention
d) Trench fever
53. Incidence can be calculated in
45. Which of the following diseases is not a) Case-control study
included in “Vision 2020 - Right to Sight” b) Prospective study
immediate goals? c) Retrospective study
a) Cataract d) Cross-sectional study
b) Epidemic conjunctivitis
c) Onchocerciasis 54. A doctor order 6 tests for SLE . Which of
d) Trachoma the following is needed for inference?
a) Prior probability of SLE,
46. Direct standardisation is used to sensitivity and specificity of test
compare the mortality rates between b) Incidence of SLE and
two countries.this is done because of predictivity of each test
differences in c) Incidence and prevalence of
a) Causes of death SLE
b) Numerators d) Relative risk of SLE in the
c) Age distribution patient
d) Denominators
55. Mercury is disposed by
47. A test has high false positive rate in a a) Controlled combustion
community. True is b) Deep burial
a) High prevalence c) Safely collect & re-use
b) Low prevalence d) Chemical treatment
c) High sensitivity
d) High specificity 56. Which of the following is used to denote
a continuous variable?
48. Weight of Indian reference man a) Simple bar
a) 60 b) Histogram
b) 55 c) Line diagram
c) 50 d) Multiple bar
d) 45
57. Crude birth rate – NOT true is
49. Aedes-True are A/E a) It is a measure of fertility
a) Recurrent biters b) It is actually a ratio not a rate
b) Eggs can’t survive >1 wk in c) it is independent of age of
water population
c) Transmits Dengue d) Numerator does not include
d) It takes 7-8 days to develop the still births
parasite & transmit the disease
58. Savlon contains
50. Chlorine demand estimated by a) Cetrimide + chlorhexidine
a) Horrock’s apparatus b) Cetrimide + chlorhexidine +
b) Berkefield filter butyl alcohol
c) Chlorometer c) Cetrimide + butyl alcohol
d) Double pot method d) Cetrimide + Cetavlon

51. STEPS done for 59. All of the following statements about
a) Surveillance of risk factors of purification of water are true except
non-communicable disease a) Presence of Clostridial spores
b) Surveillance of incidence of indicates recent contamination
non-communicable disease of water
c) Surveillance of evaluation of b) Coliforms must not be
treatment of non- detectable in any 100 ml
communicable disease sample of drinking water
d) Surveillance of mortality from c) Sodium thiosulphate is used to
non-communicable disease neutralize certain contaminants

5
Online support at: www.saraswatastar.weebly.com
Crazy Complete Compendium of AIIMS May 2010
© Dr Saraswata Das. All Rights Reserved.

d) Coliforms may be detected by 67. Dose limiting toxicity of


multiple tube method & indole chemotherapeutic agents
production a) Bone marrow toxicity
b) GI toxicity
60. Leprosy involves A/E c) Neurotoxicity
a) Uterus d) Respiratory problems
b) Ovary
c) Nerve 68. Opioid agonist are A/E
d) Eye a) Morphine
b) Codeine
61. A pregnant female in first trimester c) Ketamine
came with sputum positive TB. d) Methadone
Treatment of choice
a) Start Cat I treatment 69. Drug NOT used in pulmonary
immediately hypertension
b) Start Cat II treatment a) CCB
immediately b) Endothelin receptor antagonist
c) Start Cat III treatment c) Alpha blocker
immediately d) Prostacyclin
nd
d) Delay treatment till 2
trimester 70. Thalidomide is used in all except
a) HIV induced peripheral
62. In a study following interpretation are neuropathy
obtained: Satisfied,Very b) HIV induced mouth ulcer
satisfied,Dissatisfied. Which type of c) Behcet's syndrome
scale is this? d) Erythema nodosum leprosum
a) Nominal
b) Ordinal 71. Oligospermia caused by
c) Interval a) Methotrexate
d) Ratio b) Hydroxychloroquine
c) Leflunomide
63. Chikungunya is transmitted by d) D-Penicillamine
a) Aedes
b) Culex 72. Which of the following drugs is both anti
c) Mansonoides resoptive and bone formative?
d) Anopheles a) Strontium ranelate
b) Calcitonin
c) Ibadronate
PHARMA d) Teriperatide

64. Which is NOT an alkylating agent? 73. Drug of choice for Zollinger-Ellison
a) 5-FU syndrome
b) Chlorambucil a) Antihistaminics
c) Melphalan b) Proton pump inhibitors
d) Cyclophosphamide c) Dopamine agonists
d) Antacids
65. Which one of the following drugs cause
hypomagnesemia by increased 74. Pharmacovigilance is done for
excretion? monitoring
a) Frusemide therapy a) Drug price
b) Cisplatin b) Unethical practices
c) Digitalis c) Drug safety
d) Aminoglycosides d) Pharmacology students

66. Which is NOT used in treatment of 75. All of the following drug is CYP3A
heroin dependence? inhibitor except
a) Disulfiram a) Erythromycin
b) Buprenorphine b) Itraconazole
c) Clonidine c) Ritonavir
d) Lofexidine d) Saquinavir

6
Online support at: www.saraswatastar.weebly.com
Crazy Complete Compendium of AIIMS May 2010
© Dr Saraswata Das. All Rights Reserved.

MICRO 84. Arbovirus – True is


a) Yellow fever is endemic in
76. Most sensitive test for Treponema India
a) VDRL b) Dengue has only one serotype
b) RPR c) KFD was first identified in West
c) FTA-ABS Bengal
d) Kahn d) Chikungunya is transmitted by
Aedes
77. A male patient presented with
granulomatous penile ulcer. On Wright 85. Lymes disease all are true except
geimsa stain tiny organisms of 2 a) Borellia burgdorferi replicates
microns within macrophages seen. locally and invades locally
What is the causative organism? b) Infection progresses inspite of
a) LGV good humoral immunity
b) Calymmatobacterium c) Polmorphonuclear
granulomatis lymphocytosis in CSF suggest
c) Neisseria meningeal involvement
d) Staph aureus d) IgA intrathecally confirms
meningitis
78. Maltese cross seen on polarizing
microscopy in 86. Streptococcus all are true except
a) Cryptococcus neoformans a) Streptodornase cleaves dna
b) Penicillium marneffi b) Streptolysin O is active in
c) Blastomyces reduced state
d) Candida albicans c) Streptokinase is produced from
serotype strep. A,C,K
79. Diagnostic of Rabies d) Pyrotoxin A is plasmid
a) Guaneri bodies mediated
b) Negri bodies
c) Cowdry A body 87. Not used in leptospirosis
d) Cowdry B body a) Microscopic agglutination test
b) Dark field illumination
80. HIV pt with malabsotion, fever, chronic c) Macroscopic agglutination test
diarrohea, with acid fast positive d) Weil felix reaction
organism. What is the causative agent?
a) Giardia 88. Superantigens true is
b) Microsporidia a) Directtly attached to variable
c) Isospora region of MHC
d) E.histolytica b) Needs to proccessed and
presented to cleft of MHC
81. Congenital toxoplasmosis – False is c) Not recalled
a) Diagnosed by detection of IgM d) Not recalled
in cord blood
b) IgA is more sensitive than IgM 89. CD4+ is not important for which of the
for detection following
c) Dye test is gold standard for a) Antibody production
IgG b) Cytotoxicity of T cells
d) Avidity testing must be done to c) Memory B cells
differentiate between IgA & IgM d) Opsonisation

82. Malabsorption caused by A/E 90. Antigen antibody precipitation is


a) Giardia maximally seen in which of the
b) Ascaris following?
c) Strongyloides a) Excess of antibody
d) Capillaria philipensis b) Excess of antigen
c) Equivalence of antibody and
83. Irradiation can be used to sterilize A/E antigen
a) Bone graft d) Antigen-Hapten Interaction
b) Suture
c) Artificial tissue graft
d) Bronchoscope

7
Online support at: www.saraswatastar.weebly.com
Crazy Complete Compendium of AIIMS May 2010
© Dr Saraswata Das. All Rights Reserved.

PATHO b) Papillary CA of thyroid


c) Serous cystadenocarcinoma of
91. Coagulative necrosis seen in ovary
a) TB d) Meningioma
b) Sarcoidosis
c) Cryptococcal infection 100. Central organ in apoptosis
d) Gangrene a) Mitochondria
b) Nucleus
92. Characteristic feature of acute c) ER
inflammation d) Golgi body
a) Vasoconstriction
b) Vascular stasis 101. CD marker specific for myeloid series
c) Vasodilatation and increased a) CD34
vascular permeability b) CD45
d) Margination of leucocytes c) CD99
d) CD117
93. Berry aneurysm – Defect lies in
a) Degeneration of internal elastic 102. Two siblings with osteogenesis
lamina imperfecta.. but their parents are
b) Degeneration of media / normal. Mechanism of inheritance is
muscle cell layer a) Anticipation
c) Deposition of mucoid material b) Genomic imprinting
in media c) Germ line mosaicism
d) Low grade inflammation of d) New mutation
vessel wall
103. Not true about xanthogranulomatous
94. Which of the following is associated with inflammation
aging a) Associated with TB
a) Reduced cross linkages in b) Yellow granuloma
collagen c) Giant cells seen
b) Increased free radical injury d) Foam cells seen
c) Somatic mutations in DNA
d) Increased superoxide 104. All of the following genes are implicated
dismutase levels in Carcinogenesis of colon except
a) APC
95. Caspases are associated with b) k-ras
a) Organogenesis c) Beta catenin
b) Hydropic degeneration d) Mismatch repair
c) Collagen hyalinization
d) Not recalled
105. Wilm’s tumor associated with A/E
96. Smoking is a causative factor for all the a) Hemihypertrophy
carcinomas except b) Aniridia
a) Esophageal carcinoma c) Hypertension
b) Laryngeal carcinoma d) Bilateral polycystic kidney
c) Nasopharyngeal carcinoma
d) Urinary bladder carcinoma
106. Which of the following is NOT true about
97. Barrett’s esophagus shoes FAP?
a) Intestinal dysplasia a) AR inheritance
b) Intestinal metaplasia b) Screening done by
c) Squamous cell metaplasia sigmoidoscopy
d) Columnar cell metaplasia c) Polyps develop in late
adulthood
98. Hypersensitivity vasculitis affects d) Epidermal cysts & osteomas
a) Post-capillary venules may occur
b) Arterioles
c) Veins
d) Medium-sized arteries MEDICINE

99. Psammoma bodies seen in A/E 107. Hyperextensibility with normal elastic
a) Follicular CA of thyroid recoil is a feature of

8
Online support at: www.saraswatastar.weebly.com
Crazy Complete Compendium of AIIMS May 2010
© Dr Saraswata Das. All Rights Reserved.

a) Ehlers Danlos syndrome a) HBV


b) Pseudoxanthoma elasticum b) HCV
c) Cutis laxa c) EBV
d) Scleroderma d) Parvovirus

108. Which of the following about 115. Rapid infusion of insulin causes
atherosclerosis is true? a) Hyperkalemia
a) Intake of PUFA associated with b) Hypokalemia
decreased risk c) Hypernatremia
b) Thoracic aorta involvement is d) Hyponatremia
more severe than abdominal
aorta involvement 116. Digitalis toxicity enhanced by A/E
c) Extent of lesion in veins is a) Renal failure
same as that in arteries b) Hyperkalemia
d) Hypercholesterolemia does not c) Hypercalcemia
always increase the risk of d) Hypomagnesemia
atherosclerosis per se
117. True about hemochromatosis
109. Not a cardiovascular complication of a) Is genetically heterogenous
HIV/AIDS b) Cannot be treated by
a) Cardiac tamponade phlebotomy
b) Recurrent arterial embolism c) Completely penetrant
c) CHF d) More common in female than
d) Aortic aneurysm male

110. A girl presented with severe 118. Strawberry gingivitis seen in


hyperkalemia and peaked T waves on a) Myelocytic infiltration
ECG. Fastest way of shifting potassium b) Phenytoin toxicity
intracellularly is c) Wegener granulomatosis
a) Calcium gluconate IV d) Klippel Trenaunay syndrome
b) Oral resins
c) Insulin + glucose 119. A child presented with arthritis and
d) Sodium bicarbonate purpura. Laboratory examination
showed monoclonal and polyclonal
111. Features of hypocalcemia are A/E cryoglobulins. Histopathology showed
a) Numbness & tingling deposits of cryoglobulins around the
b) Circumoral paresthesia vessels. The patient should be tested for
c) Depressed tendon reflexes which of the following?
d) Skin irritability & sensitivity a) C3
b) C5a
112. A young man back from leisure trip has c) C1 esterase inhibitor
swollen knee joints & foriegn body d) C9
sensation in eyes. Llikely cause is
a) Reiter’s syndrome 120. Test for assessment of absorptive
b) Sarcoidosis function of colon
c) Behcet’s disease a) D-xylose test
d) SLE b) NBT-PABA test
c) Fecal fat estimation
113. Cause of vasodilatation in spider nevi d) Schilling test
a) Estrogen
b) Testosterone 121. All are true about temporal arteritis
c) Hepatotoxins except
d) FSH a) Can lead to sudden bilateral
blindness
114. A patient presented with arthritis and b) More common in females
purpura. Laboratory examination c) Worsens on exposure to heat
showed monoclonal and polyclonal d) Mostly affects elderly
cryoglobulins. Histopathology showed
deposits of cryoglobulins around the 122. A pt presents with acute rheumatic
vessels. The patient should be tested for carditis with fever. True statement is
which of the following? a) Increased troponin T

9
Online support at: www.saraswatastar.weebly.com
Crazy Complete Compendium of AIIMS May 2010
© Dr Saraswata Das. All Rights Reserved.

b) Reduced myocardial 129. 6 weeks old female baby found


contractility unconscious suddenly in the crib. She
c) Signs of inflammation and was previously healthy. Normal blood
necrosis pressure, hyperpigmentation of genitals,
d) Valve replacement will blood glucose 30 mg/dl. Diagnosis is
ameliorate CCF a) CAH due to 21-alpha
hydroxylase deficiency
b) Familial glucocorticoid
deficiency
PEDIATRICS c) Cushing’s syndrome
d) Insulinoma
123. All are signs of impending Eisenmenger
except 130. Best indicator of growth monitoring in
a) Increased flow murmur across children
tricuspid & pulmonary valve a) Weight
b) Single S2 b) Mid-arm circumference
c) Loud P2 c) Rate of increase of height &
d) Graham Steele murmur weight
d) Head circumference
124. Eisenmenger syndrome – True are A/E
a) Pulmonary veins are not 131. A child presents with failure to thrive
distended with frequent vomiting,
b) RV & LV walls come back to diarrhoea,hepatosplenomegaly &
normal size abdominal distension. CT shows
c) Dilatation of central pulmonary adrenal calcification. Which of the
artery following is the diagnosis?
d) Peripheral pruning of a) Adrenal hemorrhage
pulmonary arteries b) Wolman’s disease
c) Pheochromocytoma
d) Addison’s disease
125. Chang staging is used for
a) Retinoblastoma 132. Most sensitive indicator of intravascular
b) Medulloblastoma volume depletion in infant
c) Ewing’s sarcoma a) Stroke volume
d) Rhabdomyosarcoma b) Heart rate
c) Cardiac output
126. Which is the most common CNS tumor d) Blood pressure
to metastasize outside the brain?
a) Glioblastoma multiforme 133. An alert 6 month old child is brought
b) Medulloblastoma with vomiting & diarrhea. RR-45/min,
c) Ependymoma HR-180/min, SBP-85 mm of Hg.
d) Choroid plexus tumor Extremities are cold & mottled. Capillary
refilling time is 4 secs. Diagnosis is
127. Which of the following is a component of a) Early compensated
Pentalogy of Fallot hypovolemic shock
a) ASD b) Early decompensated
b) PDA hypovolemic shock
c) Coarctation of aorta c) Late compensated
d) LVH hypovolemic shock
d) Late decompensated shock
128. A newborn has been brought with due to SVT
seizures refractory to treatment and a
continuous bruit through the anterior
fontanelle. CT shows midline lesion with SURGERY
hypoechogenesity and dilated lateral
ventricles. Diagnosis is 134. Most sensitive screening procedure for
a) Vein of Galen malformation CA Prostate
b) Arachnoid cyst a) DRE
c) Teratoma b) PSA
d) Encephalocele c) DRE + PSA
d) Endorectal coil MRI with T1W &
T2W images

10
Online support at: www.saraswatastar.weebly.com
Crazy Complete Compendium of AIIMS May 2010
© Dr Saraswata Das. All Rights Reserved.

b) Right medial visceral rotation


135. Orchiopexy for undescended testes is c) Cranial visceral rotation
done ideally at what age? d) Caudal visceral rotation
a) Neonate
b) 1-2 yrs 143. Best prognostic factor for head injury
c) 5 yrs a) Age
d) Puberty b) Mode of injury
c) Glasgow coma scale
136. Polyp not associated with risk of d) CT finding
malignancy
a) Juvenile polyp 144. Most immediate complication of
b) FAP ileostomy in the post-operative period
c) Peutz Jeghers syndrome a) Obstruction
d) Juvenile polyposis syndrome b) Necrosis
c) Diarrhea
137. Peutz Jeghers polyps present in d) Prolapse
a) Rectum
b) Colon 145. Follicular carcinoma differs from
c) Esophagus follicular adenoma by
d) Jejunum a) Vascular invasion
b) Increased mitotic figures
138. Renal calculi associated with Proteus c) Hurthle cell change
infection d) Lining of columnar & cuboidal
a) Uric acid cells
b) Triple phosphate
c) Calcium oxalate 146. Hashimoto’s thyroiditis – True are A/E
d) Xanthine a) Follicular destruction
b) Increase in lymphocytes
139. Congenital hypertrophic pyloric stenosis c) Oncocytic metaplasia
associated with d) Orphan Annie eye nuclei
a) Hypokalemic acidosis
b) Hypokalemic alkalosis 147. Most reliable investigation for
c) Hyperkalemic acidosis Amyloidosis
d) Hyperkalemic alkalosis a) Rectal biopsy
b) Sigmoidoscopy
140. A man with blunt abdominal trauma c) Colonoscopy
with h/o pelvic fracture has presented to d) Gingival biopsy
ER. He has passed only few drops of
blood per meatus and no urine in the 148. A 60 yr old chronic smoker presents
past 8 hrs. His bladder is palpable per with painless gross hematuria of 1 day
abdomen. Which of the following is duration. Investigation of choice to know
correct? the cause of hematuria
a) Anuria due to hemorrhagic a) USG
shock b) X-ray KUB
b) Urethral injury c) Urine routine microscopy
c) Ureteral rupture leading to d) Urine cytology for malignant
extravasation of urine in cells
retroperitoneum
d) Bladder rupture 149. In blunt trauma abdomen what should
be the approach for doing laparotomy?
141. Sister Mary Joseph nodule associated a) Depends on organ injured
with b) Always midline
a) Pancreatic ca c) Always transverse
b) Gastric ca d) Depends upon type of injury
c) Colonic ca
d) Ovarian ca 150. Which of the following is true about
branchial anomalies?
nd
142. In order to expose the celiac axis, left a) Mostly arises from 2
renal artery, superior mesenteric artery branchial arch
& abdominal aorta in a case of trauma, b) All patients present with stridor
which of the following is required? at birth
a) Left medial visceral rotation

11
Online support at: www.saraswatastar.weebly.com
Crazy Complete Compendium of AIIMS May 2010
© Dr Saraswata Das. All Rights Reserved.

c) Branchial cyst is more common b) Follows Non-Newtonian fluid


than sinus kinetics
d) All branchial sinuses need to c) Contains hyaluronic acid
be excised d) Viscosity is variable

ORTHO GYNAE & OBS

151. A 68 yr old man came with pain and 158. Pain of ovarian carcinoma is referred to
swelling of right knee. Ahlbeck grade 2 a) Back of thigh
osteoarthritic changes were found on b) Gluteal region
investigation. What is the further c) Anterior surface of thigh
management? d) Medial surface of thigh
a) Conservative
b) Arthroscopic washout 159. A woman with primary infertility with CA-
c) High tibial osteotomy 125 level 90 iu. She has bilateral
d) Total knee replacement palpable mass. USG shows mass in
pelvis. Diagnosis may be
152. Which of the following is a pulsatile a) Ovarian CA
tumor? b) TB
a) Osteosarcoma c) Endometrioma
b) Chondrosarcoma d) Borderline ovarian tumor
c) Osteoclastoma
d) Ewing’s sarcoma 160. Causes of primary amenorrhea are A/E
a) Rokitansky syndrome
153. Posterior glenohumeral instability is b) Sheehan syndrome
tested by c) Kallman syndrome
a) Jerk test d) Turner syndrome
b) Crank test
c) Sulcus test 161. Test for ovarian reserve
d) Fulcrum test a) LH
b) LH/FSH ratio
154. Mineral of the bone is c) FSH
a) Calcite d) Estradiol
b) Hydroxyapatite
c) Calcium oxide 162. Clomiphene citrate – True statement is
d) Calcium carbonate a) Euclomiphene is anti-
estrogenic
155. 65 yrs old man with H/o of back pain b) Pregnancy rate is 3 times as
since 3 months. ESR is raised. Marked compared to placebo
stiffness on examination. Mild restriction c) Incidence of twins is 5-6%
of chest movements. On X-ray, d) It has been shown to increase
syndesmophytes are present. Diagnosis fertility in oligospermic males
is
a) Ankylosing spondylitis 163. At what gestational age should a
b) Degenerative osteoarthritis of pregnancy with cholestasis of
spine pregnancy be terminated?
c) Ankylosing hyperosteosis a) 34 weeks
d) Lumbar canal stenosis b) 36 weeks
c) 38 weeks
156. A lady presents with right knee swelling. d) 40 weeks
Aspiration was done in which CPPD
crystals were obtained. Next best 164. A lady with placenta previa delivered a
investigation is baby. She had excessive bleeding. After
a) ANA resuscitation most likely complication is
b) RF a) Galactorrhoea
c) CPK b) Diabetes insipidus
d) TSH c) Loss of menstruation
d) Cushing’s syndrome
157. Synovial fluid – True statements are
A/E
a) Secreted by Type A cells

12
Online support at: www.saraswatastar.weebly.com
Crazy Complete Compendium of AIIMS May 2010
© Dr Saraswata Das. All Rights Reserved.

165. Which of the following treatments for d) Butyrylcholinesterase


menorrhagia is not supported by
evidence? 173. A pregnant lady had no complaints but
st
a) Tranexemic acid mild cervical lymphadenopathy in 1
b) Ethamsylate trimester. She was prescribed
c) Combined OCP spiramycin but she was non compliant.
d) Progesterone Baby was born with hydrocephalus &
intracerebral calcification. Which of
166. Vaginal delivery can be allowed in A/E these is likely cause
a) Monochorionic monoamniotic a) Toxoplasmosis
twins b) CMV
b) Mento-anterior c) Cryptococcus
c) Extended breech d) Rubella
d) Dichorionic twins – First vertex,
second breech
OPHTHAL
167. A 35 yr old P3+0 is observed to have CIN
grade III on colposcopic biopsy. Best 174. Which of the following does NOT
treatment will be scavenge free radicals in the lens?
a) Cryosurgery a) Vit A
b) Conization b) Vit C
c) LEEP c) Vit E
d) Hysterectomy d) Catalase

168. Lady undergoes total radical 175. Highest refractive index


hysterectomy for Stage Ib Ca Cervix. It a) Cornea
is found that Ca extends to lower part of b) Anterior surface of lens
body of uterus and upper part of cervix. c) Posterior surface of lens
Next step of management d) Centre of lens
a) Chemotherapy
b) Radiotherapy 176. Mucopolysaccharide hyaluronic acid is
c) Chemoradiotherapy present in
d) Follow-up a) Aqueous humor
b) Vitreous humor
169. All are seen in gestational diabetes c) Lens
except d) Cornea
a) Previous macrosomic baby
b) Obesity 177. All are important causes of childhood
c) Malformations blindness in India except?
d) Polyhydramnios a) Glaucoma
b) Congenital dacryocystitis
170. A pregnant female at 35 wks of c) Malnutrition
pregnancy is diagnosed with SLE. d) Ophthalmia neonatorum
Drugs that can be used to treat are A/E-
a) Corticosteroids 178. A young male presents with h/o
b) Sulphasalazine unilateral progressive proptosis for
c) Methotrexate many years. Swelling is increased on
d) Hydroxychloroquine bending forward and compressible,USG
shows retrobulbar echogenicity. What is
171. Ideal contraceptive for lactating mothers the diagnosis?
a) Barrier method a) Neurofibromatosis
b) Combined OCP b) Orbital varix
c) Lactational amenorrhoea c) Orbital A-V fistula
d) Progesterone only pills d) Orbital encephalocele

172. The presence of increased levels of 179. Which of the following ocular conditions
which of the following in amniotic fluid is is autosomal dominant in inheritance?
an indicator of open neural tube defect a) Best disease
in the fetus? b) Gyrate atrophy
a) Phosphatidylesterase c) Lawrence-Moon-Biedel
b) Acetylcholinesterase syndrome
c) Pseudocholinesterase d) Bassen Kornzweig disease

13
Online support at: www.saraswatastar.weebly.com
Crazy Complete Compendium of AIIMS May 2010
© Dr Saraswata Das. All Rights Reserved.

in the portal vein. Portal flow is


maximally reduced by
ENT a) Ether
b) Halothane
180. Most common site for CSF rhinorrhoea c) Isoflurane
a) Frontal sinus d) Enflurane
b) Ethmoidal sinus
c) Tegmen tympani 188. A young boy has sickle cell trait. which
d) Maxillary sinus of the following anaesthesia is
contraindicated?
181. A case of Bell’s palsy presents on day 3. a) IV regional anaesthesia
Which of the following is the appropriate b) Brachial plexus block by
management? suraclavicular approach
a) Oral steroids alone c) Brachial plexus block by
b) Oral steroids + antiviral infraclavicular approach
acyclovir d) Brachial plexus block by
c) Acyclovir + amantadine axillary approach
d) Intralabyrinthine steroids
189. Methemoglobinemia caused by
182. Endolymph in inner ear produced by a) Procaine
a) Phalangeal cells b) Prilocaine
b) Stria vascularis c) Bupivacaine
c) Lamina spiralis ossea d) Lignocaine
d) Basilar membrane

183. Extrinsic membranes of larynx are A/E RADIO


a) Cricothyroid
b) Thyrohyoid 190. A 48 yr old woman comes with b/l
c) Hyoepiglottic progressive weakness of both lower
d) Cricotracheal limbs, spasticity & mild impairment of
respiratory movements. MRI shows an
184. Infection of CNS spreads in inner ear intradural mid-dorsal midline enhancing
through lesion. What is the diagnosis?
a) Cochlear aqueduct a) Intradural lipoma
b) Endolymphatic sac b) Meningioma
c) Vestibular aqueduct c) Neuroenteric cyst
d) Hyrtle fissure d) Dermoid cyst

191. A patient brought in unconscious state


SKIN with external injuries. CT is normal with
no midline shift and ventricles are
185. Which of the following is NOT a primary normal. Multiple small hemorhages are
skin disease? seen at the base of the brain. Diagnosis
a) Reiter’s syndrome is
b) Bowen’s disease a) Subarachnoid hemorrhage
c) Psoriasis b) Diffuse axonal injuries
d) Lichen planus c) Concussion
d) Contusion
186. 25 yr old male having fever & malaise
since 2 weeks, arthritis of ankle joint and 192. Investigation of choice for a lesion of
tender erythematous nodules over the temporal bone
shin. Diagnosis is a) CT
a) Erythema nodosum b) MRI
b) Hensen’s disease c) USG
c) Weber-Christian disease d) Plain X-ray
d) Urticarial vasculitis
193. Which of the following feature of thyroid
nodule on usg is not sugestive of
ANESTHESIA malignancy?
a) Hyperechogenesity
187. The volatility of an anesthetic agent is b) Hypoechogenesity
directly proportional to lowering the flow c) Non-homogenous

14
Online support at: www.saraswatastar.weebly.com
Crazy Complete Compendium of AIIMS May 2010
© Dr Saraswata Das. All Rights Reserved.

d) Microcalcification

194. Radiological features of left ventricular Question Recall Acknowledgement


failure are all except
a) Oligemic lung fields Special Thanks to PrePgMedicos
b) Increased flow in upper lobe
veins Unique Question Recaller
c) Cardiomegaly
d) Kerley B-lines

195. Characteristic radiological feature of


fibrous dysplasia
a) Thickened bone matrix
b) Cortical erosion
c) Ground glass appearance
d) Bone enlargement

PSYCHIATRY

196. Naltrexone is used in opioid


dependence to
a) Prevent respiratory depression
b) Treat withdrawl symptoms
c) Prevent relapse Read the “Answers &
d) Detoxification of opioid
overdose Explanations” part to
197. Drug of choice in OCD clear your confusions!
a) Fluoxetine
b) Imipramine
c) Alprazolam
d) Chlorpromazine

198. 3 yr old girl has delayed devolpmental


milestone, delayed speech, difficulty in
concentrating on studies, plays with
herself & does not make friends.
Diagnosis is
a) Autism
b) ADHD
c) Specific learning disorder
d) Mental retardation

199. Most common substance abuse in India


a) Tobacco
b) Cannabis
c) Alcohol
d) Opium

200. A young lady presents with repeated


episodes of excessive eating followed
by purging by use of laxatives. What is
the diagnosis?
a) Binge eating
b) Bulimia nervosa
c) Schizophrenia
d) Anorexia nervosa

15
Online support at: www.saraswatastar.weebly.com
Crazy Complete Compendium of AIIMS May 2010
© Dr Saraswata Das. All Rights Reserved.

AIIMS May 2010 nerve innervating the premolars and


Answers & Explanations molars alone.

Please Note: For the controversial questions, Maxillary nerve traverses the infraorbital
the BEST POSSIBLE answer has been groove and canal in the floor of the orbit,
provided after going through standard and appears upon the face at the
references. However, these are not absolute & infraorbital foramen. Here is it referred
are open for discussion. Repeat questions to as the infraorbital nerve, a terminal
have NOT been explained for obvious branch. At its termination, the nerve lies
reasons. beneath the quadratus labii superioris,
and divides into a leash of branches
ANATOMY which spread out upon the side of the
nose, the lower eyelid, and the upper lip,
1. C (Anterior nasal division of maxillary joining with filaments of the facial nerve.
nerve)

The middle superior alveolar nerve is Inferior alveolar nerve is a branch of


a nerve that “Middle superior alveolar Mandibular division of Trigeminal nerve.
nerve arises from the infraorbital nerve So options A & D are outrightly wrong.
(a continuation of Maxillary nerve) as it
lies in the maxillary groove. It descends
2. D (Levator scapulae)
in the lateral wall of the maxillary sinus
& supplies the upper premolar teeth &
the adjoining parts of the gum & cheek. Rhomboid major, Rhomboid minor &
Trapezius help in both elevation &
retraction of the scapula.
The middle superior alveolar nerve is a
nerve that drops from the infraorbital
portion of the maxillary nerve to supply Levator scapulae helps only in
elevation of the scapula. It has no role
the sinus mucosa, the roots of the
maxillary premolars, and the in retraction.
mesiobuccal root of the first maxillary
molar. It is not always present; and in Ref: Gray’s Anatomy for Students, 1e,
the majority of cases it is non existent Table 7.1, p.634
with the posterior superior alveolar

16
Online support at: www.saraswatastar.weebly.com
Crazy Complete Compendium of AIIMS May 2010
© Dr Saraswata Das. All Rights Reserved.

3. A (Trochlear) granules contain several other enzymes


other than lysozyme.
Oculomotor, Facial & Glossopharyngeal
nerves carry GVE (General Visceral This is a controversial question. B
Efferent) fibers that are part of appears slightly better than D to me.
parasympathetic division of ANS. However, other opinions are welcome.
This question is open for discussion.
Trochlear nerve carries only GSE
(General Sensory Efferent) fibers to 9. A
innervate the superior oblique muscle. It
is the only cranial nerve to exit from 10. D
the posterior surface of the
brainstem. 11. A (Maxillary sinus)

4. A (Repeat from AIIMS Nov 09, AI 2010) The anterior ethmoidal nerve travels with
the anterior ethmoidal artery and leaves
5. B the orbit through a canal between the
ethmoidal labyrinth and the frontal bone. It
6. C (Repeat from AI 2010) passes through and supplies the adjacent
ethmoidal cells and frontal sinus, and then
7. A (Anterolateral & around the aorta) enters the cranial cavity immediately
lateral and superior to the cribriform plate.
Celiac plexus is the large accumulation
of nerve fibers & ganglia associated with The anterior ethmoidal nerve travels
the roots of the celiac trunk & superior forward in a groove on the cribriform plate
mesenteric artery immediately below the and then enters the nasal cavity by
aortic hiatus of the diaphragm. descending through a slit-like foramen
immediately lateral to the crista galli. It has
branches to the medial and lateral wall of
the nasal cavity and then continues
forward on the undersurface of the nasal
bone. It passes onto the external surface
of the nose by traveling between the nasal
bone and lateral nasal cartilage, and then
terminates as the external nasal nerve,
which supplies skin around the naris, in
the nasal vestibule, and on the tip of the
nose.

Ref: Gray’s Anatomy for Students, 1e,


p.980

Anterior ethmoidal nerve - A nerve that


innervates the anterior cranial fossa, the
nasal cavity, and the skin of the lower half
of the nose.

Ref: Mosby's Medical Dictionary, 8th edition. ©


2009, Elsevier.

8. B(Rich in Zinc)(Based on AI09–Q.17 MK)

Paneth cells are rich in Zinc & secrete


lysozyme. However, apart from
lysozyme, they secrete other proteins
also like peptidase, TNF-alpha and
defensins or cryptidins. So the secretory

17
Online support at: www.saraswatastar.weebly.com
Crazy Complete Compendium of AIIMS May 2010
© Dr Saraswata Das. All Rights Reserved.

12. A (Movement of tibial tuberosity inflammatory stimulus in the direction of the


towards lateral border of patella) gradient of locally produced chemoattractants.
th
- Robbins Pathologic Basis 8 ed. E-book;
th
When the foot if firmly planted on the 7 /56
ground when a person is standing, the
femur is medially rotated on the tibia to BIOCHEMISTRY
lock & stabilize the knee joint. However,
if the foot is raised off the ground, the 24. B
tibia may be laterally rotated on the
femur to lock the knee joint 25. B

Ref: Clinical Anatomy – Richard S. 26. A


Snell, 7e, p.680 Footnote.
27. C
13. C (Repeat from AI 2009 – Q.13 MK)
28. D (Repeat from AIIMS May 09 – Q.21 AA)
14. D (Sigmoid colon)
29. B (Repeat from AIIMS May 09 – Q.24 AA)
Epiploic appendix - one of the fat
pads, 2 to 10 cm long, scattered through 30. C (Repeat from AI 2009 – Q.48a MK)
the peritoneum along the colon and the
upper part of the rectum, especially 31. A (Repeat from AI 2009 – Q.166 MK)
along the transverse and sigmoid colon.
32. B (Alkaptonuria)
Ref: Mosby's Medical Dictionary, 8th
edition. © 2009, Elsevier
Clinical manifestations of alcaptonuria consist
So although they are found throughout of ochronosis and arthritis, which occur in
the colon including cecum & upper part adulthood. The only sign of the disorder in
of the rectum, sigmoid colon appears to children is a blackening of the urine on standing.
be the best option here. This is caused by oxidation and polymerization of
the homogentisic acid. Urine with acid pH may
15. A (Repeat from AIIMS May 09 – Q.4 AA) not darken even after many hours. This sign may
never be noted, delaying the diagnosis until
adulthood. Ochronosis, seen clinically as dark
PHYSIOLOGY spots on the sclera or ear cartilage, results from
the accumulation of the black polymer of
16. C (Repeat from AI 2009 – Q.36 MK) homogentisic acid. Arthritis can be disabling and
occurs in almost all affected subjects with
17. A (Repeat from AIIMS May 09 – Q.13 AA) advancing age. It involves the large joints (spine,
hip, and knee) and is usually more severe in
18. B (Repeat from AI 2009 – Q.28 MK) males. Like rheumatoid arthritis, the arthritis has
acute exacerbations, but the radiologic findings
19. B (Repeat from AI 2009 – Q.31 MK) are typical of osteoarthritis, with characteristic
narrowing of the joint spaces and calcification of
20. A (Repeat from AIIMS May 09 – Q.20 AA) the intervertebral discs. High incidences of heart
disease (mitral and aortic valvulitis, calcification
21. D (Repeat from AI 2009 – Q.167 MK) of the heart valves, and myocardial infarction)
have been noted.
22. D (Repeat from AIIMS May 09 – Q.18 AA)

23. A Ref: Nelson’s Pediatrics, 18e, P.534

Chemotaxis of Leukocytes. Please Note:


After exiting the circulation, leukocytes
emigrate in tissues toward the site of injury by This question seems to be framed based on a
a process called chemotaxis, which is defined Case Discussion in the Indian Journal of Clinical
as locomotion oriented along a chemical Biochemistry, 2007 / 22 (2) 158-161.
gradient. Both exogenous and endogenous
substances can act as chemoattractants. The This article is freely available on the net (link
net result is that leukocytes migrate toward the given below). It even has photographs of a 45-yr
old female patient showing the ochronotic
changes & the X-ray showing calcification of

18
Online support at: www.saraswatastar.weebly.com
Crazy Complete Compendium of AIIMS May 2010
© Dr Saraswata Das. All Rights Reserved.

intervertebral disc spaces. Please go through it early symptom. Older untreated children become
once, it will help to clear your concepts. hyperactive, with purposeless movements,
rhythmic rocking, and athetosis.
Ref: http://medind.nic.in/iaf/t07/i2/iaft07i2p158.pdf
On physical examination, these infants are
lighter in their complexion than unaffected
siblings. Some may have a seborrheic or
eczematoid rash, which is usually mild and
disappears as the child grows older. These
children have an unpleasant odor of
phenylacetic acid, which has been described as
musty or mousey. There are no consistent
findings on neurologic examination. Most infants
are hypertonic with hyperactive deep tendon
reflexes. About 25% of children have seizures,
and more than 50% have
electroencephalographic abnormalities.
Microcephaly, prominent maxilla with widely
spaced teeth, enamel hypoplasia, and growth
retardation are other common findings in
untreated children. The clinical manifestations of
classic PKU are rarely seen in those countries in
which neonatal screening programs for the
detection of PKU are in effect.

TYROSINEMIA TYPE II (RICHNER-HANHART


SYNDROME,OCULOCUTANEOUS
TYROSINEMIA).
This rare autosomal recessive disorder is caused
by deficiency of tyrosine aminotransferase
enzyme, which results in palmar and plantar
hyperkeratosis, herpetiform corneal ulcers, and
mental retardation. Ocular manifestations of
Benedicts test gave dark supernatant, initial excessive tearing, redness, pain, and
greenish brown precipitate followed by a photophobia often occur before skin lesions.
yellow precipitate of cuprous oxide on Corneal lesions are presumed to be due to
standing. tyrosine deposition. In contrast to herpetic ulcers,
corneal lesions in tyrosinemia type II stain poorly
with fluorescin and often are bilateral. Skin
About other options: lesions, which may develop later in life, include
painful, nonpruritic hyperkeratotic plaques on the
CLASSIC PHENYLKETONURIA (PKU). soles, palms, and fingertips. Mental retardation,
which occurs in <50% of patients, is usually mild
to moderate.
Deficiency of the enzyme phenylalanine
hydroxylase or of its cofactor tetrahydrobiopterin
ARGININOSUCCINATE LYASE (AL)
causes accumulation of phenylalanine in body
DEFICIENCY (ARGININOSUCCINIC ACIDURIA)
fluids and the central nervous system (CNS).
Severe hyperphenylalaninemia (plasma
phenylalanine levels >20 mg/dL), if untreated, The severity of the clinical and biochemical
invariably results in the development of signs and manifestations varies considerably. In the
symptoms of classic PKU, except in rare neonatal form, signs and symptoms of severe
unpredictable occasions. hyperammonemia develop in the 1st few days of
life and mortality is usually high. Infants who
survive the 1st acute episode follow the clinical
The affected infant is normal at birth. Mental
course of the subacute form. In the subacute or
retardation may develop gradually and may not
late form, the major finding is mental retardation,
be evident for the 1st few months. It is usually
which is associated with failure to thrive and
severe, and most patients require institutional
hepatomegaly. Abnormalities of the hair
care if the condition remains untreated.
characterized by dryness and brittleness are of
Vomiting, sometimes severe enough to be
special diagnostic value. Gallstones have been
misdiagnosed as pyloric stenosis, may be an

19
Online support at: www.saraswatastar.weebly.com
Crazy Complete Compendium of AIIMS May 2010
© Dr Saraswata Das. All Rights Reserved.

seen in some of the survivors. Acute attacks of


severe hyperammonemia commonly occur during 53. B
a catabolic state.
54. A

55. C
FORENSIC 56. B

33. A 57. B

34. A Crude Birth Rate is the simplest indicator


of fertility & is defined as “the number of
35. D live births per 1000 estimated mid year
population, in a given year.” It is a rate, not
The base of skull is divided into two a ratio.
halves, each moving independently of
each other like a “hinge”, the so-called 58. A
th
“Motorcyclist’s fracture”. – Reddy 28 /247
The active ingredients in Savlon cream are
36. A Cetrimide and Chlorhexidine Gluconate. It
is commonly used for treatment of mild
37. B scarring, some mild burns, cuts and
bruises as well as skin healing treatment
38. A for spots (i.e. pimples).

SPM In India Savlon is an antiseptic liquid


marketed by Johnson & Johnson. It
39. A contains I.P. 1.5% v/v Chlorhexidine
Gluconate Solution and B.P. Strong
Stratosphere ozone layer is protective cetrimide solution eq. to Cetrimide I.P.
against UV-rays. CFC 3.0% w/v.
(chlorofluorocarbons) deplete stratosphere
ozone layer. Kyoto protocol called for 59. A (Repeat from AI 2009 – Q.137 MK)
5.2%reduction in greenhouse effect.
Presence of Clostridial spores indicate
40. B contamination in remote past.
41. A
60. A
42. A
“The most pronounced changes of gynecologic
43. B leprosy seem to occur in the ovary, followed
by the cervix, myometrium, endometrium &
44. B tube in that order.” – Gynecologic Aspects of
Leprosy – Bonar & Rabson [Reference Courtesy:
45. B Dr.Abhishek (abhi_01)]
You can download this journal article from my
46. C website.http://saraswatastar.weebly.com/controversi
al-questions.html
47. B
Counterview
This is a controversial question. Some authors
48. A
have opined that ovaries are the least affected
in leprosy. IAMS Notes also suport this. Ref:
49. B Comprehensive Textbook of Dermatolgy – Sardhana
& Garg/ 71 (Courtesy:Dr Jaya Krishna)
50. A

51. A “Common sites which are involved – Skin,


Peripheral nerve, generally involves every
52. B organ except CNS & Ovaries.”–IAMS Skin Notes

20
Online support at: www.saraswatastar.weebly.com
Crazy Complete Compendium of AIIMS May 2010
© Dr Saraswata Das. All Rights Reserved.

61. A of opioids so that higher doses produce no


greater effects. The second form of help for
62. B patients undergoing heroin detoxification is the
use of a drug, such as clonidine (Catapres), that
63. A blocks some of the withdrawal symptoms. There
is also a new method of heroin detoxification
called ultra-rapid opioid detoxification under
anesthesia/sedation, and there is an
experimental method using a medication called
PHARMA lofexidine.

64. A Clonidine is used much more frequently than


methadone in detoxification. Methadone is used
65. A frequently as long-term maintenance treatment
for heroin addiction.
Several drugs, such as loop diuretics
(including furosemide, bumetanide, LOFEXIDINE. Lofexidine is approved for use in
and ethacrynic acid), produce large increases England for opioid detoxification. It appears to
in magnesium excretion through the inhibition cause less sedation and fewer cases of low blood
of the electrical gradient necessary for pressure than clonidine. In the United States, the
magnesium reabsorption in the TAL. Long-term National Institute of Drug Abuse (NIDA) is
thiazide diuretic therapy also may cause conducting studies on this drug.
magnesium deficiency. Chronic thiazide
administration has enhanced magnesium If you still have any doubt about the answer to
excretion and has specifically reduced renal this question, just visit the following link & your
expression levels of the epithelial magnesium delusions will surely vanish!
channel TRPM6. Many nephrotoxic drugs,
including aminoglycoside antibiotics, Read more at http://www.minddisorders.com/Del-
cisplatin, amphotericin B, cyclosporine, and Fi/Detoxification.html
pentamidine, can produce urinary magnesium
wasting by a variety of mechanisms, some
of which are still unknown. For instance, 67. A
tacrolimus causes hypomagnesemia through
down-regulation of TRPM6 channels.
46 “Depression of bone marrow results in
granulocytopenia, agranulocytosis,
On the other side, aminoglycosides are thrombocytopenia, aplastic anemia. This is the
thought to induce the action of the CaSR on most serious toxicity; often limits the dose that
the TAL and DCT, producing magnesium can be employed. Infections & bleeding are the
th
wasting. Some data suggest that magnesium usual complications.” – KDT 6 /820
loss associated with cisplatin treatment is
mainly the result of lowered intestinal 68. C
absorption rather than, as presently thought,
the result of increased renal elimination. Although Ketamine interacts with opioid
Ref: receptors, it does not have any agonistic activity.
http://emedicine.medscape.com/article/246366 The following study has proved it.
-overview
PURPOSE: Ketamine is known to interact with
66. A opioid receptors. However, because this agent
does not produce opioid-like respiratory
Heroin detoxification depression, it might not interact with mu(2) opioid
Patients with heroin dependence may receive receptors. Therefore, we have studied the
help with their detoxification in one of two forms. interaction of ketamine with mu(2) opioid
Opioid agonists are drugs that act like heroin in receptors expressed in SH-SY5Y cells.
the patient's body but do not provide the same CONCLUSION: The present study indicates that
"high," and are given in gradually decreasing a clinically relevant concentration of ketamine
doses. Because these medications "act" like interacts with mu(2) opioid receptors. However,
heroin, the person does not experience no agonist activity was observed.
withdrawal symptoms. Some examples of this
kind of medication are methadone and levo- Ref: J Anesth. 1999;13(2):107-9
alpha-acetylmethadol (LAAM); buprenorphine is http://www.ketamine.com/mu2.html
a partial opioid agonist, which means that it acts
like heroin or methadone, but it limits the effects 69. C

21
Online support at: www.saraswatastar.weebly.com
Crazy Complete Compendium of AIIMS May 2010
© Dr Saraswata Das. All Rights Reserved.

http://pt.wkhealth.com/pt/re/eira/abstract.00002018-
Some of the most commonly used drugs 200124030-
prescribed to treat pulmonary hypertension
00005.htm;jsessionid=Lp8fT2gx2MfqnlMyLJkSHKphygq
include:
epoprostenol (Flolan), 3HGGvG4qP1x8zlP3vS7MNDTtL!1947086508!1811956
bosentan (Tracleer), 29!8091!-1?nav=reference
intravenous treprostinil (Remodulin), MICRO
inhaled iloprost (Ventavis),
sildenafil (Viagra, Revatio), 76. C
calcium channel blockers (such as VDRL, RPR & Kahn test are screening
nifedipine), tests. Kahn test is rarely used today.
sitaxsentan (Thelin) - not FDA approved
in the U.S., and Fluorescent Treponemal Antibody
ambrisentan (Letairis). Absorption Test is the most specific test
for syphilis. If this is positive it confirms the
Ref:http://www.medicinenet.com/pulmonar diagnosis
y_hypertension/page5.htm Ref:
http://en.wikipedia.org/wiki/Rapid_plasma_
70. A reagin

71. A 77. B

Causes of Oligospermia - chemicals and Donovanosis (Granuloma inguinale)


drugs that can inhibit gonadotropins or
interfere with spermatogenesis, such as Etiology – Calymmatobacterium
arsenic, methotrexate, granulomatis
medroxyprogesterone, nitrofurantoin,
C/F – Single ulcer, floor made up of beefy,
monoamine oxidase inhibitors, and some exuberant granulation tissue which
antihypertensives. bleeds easily on manipulation.

Ref:http://www.wrongdiagnosis.com/o/olig Tissue smear –Stained with Giemsa or


ospermia/causes.htm Leishman’s stain. Organisms appear as
intracellular (in macrophages) bipolar
72. A inclusions – safety pin or telephone
handle appearance.
73. B st
Ref: Dermatology – Neena Khanna, 1 /
238
74. C
78. A
75. D
5 causes of Maltese cross:
Like other PIs, saquinavir is metabolised by the 1. Crptococcus neoformans
cytochrome P450 (CYP) 3A4 isoenzyme and is 2. Babesia microti
3. Fabry’s ds
susceptible to interactions with inducers (e.g.
4. Paracoccidiodes
rifabutin and rifampicin) and inhibitors (e.g. 5. Cholesterol pericarditis
clarithromycin and ketoconazole) of this
enzyme. Ritonavir, nelfinavir, indinavir and
delavirdine, all CYP3A4 inhibitors, greatly 79. B
increase saquinavir plasma concentrations and
80. C
the therapeutic implications of these
interactions continue to be evaluated. While Though both Microsporidia & Isospora are
saquinavir is the least potent CYP 3A inhibitor causes of diarrhea in pts of HIV infn,
among the PIs, several drugs (notably however Microsporidia is not acid-fast.
terfenadine, astemizole and cisapride) should
not be given in combination with saquinavir. I. belli is a coccidian parasite most
commonly found as a cause of diarrhea in
patients from the Caribbean and Africa. Its
Ref: Drug Safety. 24(3):223-232, 2001 cysts appear in the stool as large, acid-fast

22
Online support at: www.saraswatastar.weebly.com
Crazy Complete Compendium of AIIMS May 2010
© Dr Saraswata Das. All Rights Reserved.

structures that can be differentiated from should be immersed in it for 10 to 30


those of cryptosporidia on the basis of minutes.
size, shape, and number of sporocysts. Ref: Disinfecting endoscopes: how not to transmit
The clinical syndromes of Isospora Mycobacterium tuberculosis by bronchoscopy.
infection are identical to those caused by Can Med Assoc J. 1980 August 23; 123(4): 275–
cryptosporidia. The important distinction is 280.
that infection with Isospora is generally
relatively easy to treat with TMP/SMX. The sterilization area is equipped with a
While relapses are common, a thrice- glutaraldehyde disinfectant container. –
weekly regimen of TMP/SMX appears Flexible Bronchoscopy By Ko Pen Wang,
adequate to prevent recurrence. Atul C. Mehta, J. Francis Turner
th
Ref: Harrison 17 / 1175 84. D

81. D 85. D

I am quoting the exact lines from Nelson Options A & B are true (see Harrison).
for each of the 4 options & the answer will Confusion lies betn options C & D.
be clear to you.
About option C
Although CSF cultures are positive in less
Option D - The avidity test can be helpful than 10% of patients with apparent
to time infection. A high-avidity test result meningitis, intrathecal antibodies and a
indicates that infection began >16 wk lymphocytic pleocytosis (approximately
earlier, which is especially useful in 100 cells/µL) are present in more than
determining time of acquisition of infection 80%. Patients with meningitis typically
in the 1st or final 16 wk of gestation. A have elevated protein concentrations (>50
low-avidity test result may be present for mg/dL) but normal glucose levels (45-80
many months and is not diagnostic of mg/dL). Oligoclonal bands specific for B
recent acquisition of infection. burgdorferi may be present.

Option A - The IgM-IFA test is useful for So option D is our answer.


the diagnosis of acute infection with T.
gondii in the older child because IgM 86. D
antibodies appear earlier, often by 5 days
after infection, and diminish more quickly Options A,B,C are true.
than IgG antibodies
87. D
Option B - The IgA-ELISA also is a
sensitive test for detection of maternal and Weil-Felix test is for Rickettsial infections.
congenital infection, and results may be
positive when those of the IgM-ELISA are 88. A
not.
Superantigens bind first to the MHC Class
Option C - The Sabin-Feldman dye test II and then coordinate to a T-cell Receptor
is sensitive and specific. It measures (TCR) with a specific variable β motif.
primarily IgG antibodies. Results should Ref:
be expressed in international units http://en.wikipedia.org/wiki/Superantigen#Binding
(IU/mL), based on international standard
reference sera available from the World 89. D
Health Organization.
Option C – “Generation of B cell memory
+
th requires CD4 T cell help and involves
Ref: Nelson 18 / 1492
triggering of CD40 in target B cells.
Several recent studies indicate that similar
82. B
signals are involved in the generation of
+
memory CD8 T lymphocytes.” – Nature
83. D
Immunology 4, 431-433 (2003)
http://www.nature.com/ni/journal/v4/n5/full/
The agent recommended for chemical
ni0503-431.html
disinfection of fibreoptic bronchoscopes is
2% glutaraldehyde solution; the instrument

23
Online support at: www.saraswatastar.weebly.com
Crazy Complete Compendium of AIIMS May 2010
© Dr Saraswata Das. All Rights Reserved.

About option B – Some CD 4+ cells have been media as it approaches the neck. At the neck of
shown to have cytotoxicity.However, perforin- the aneurysm, the muscular wall and intimal
+
positive cytotoxic CD4 T cells have been elastic lamina stop short and are absent from the
described in human immunodeficiency virus- aneurysm sac itself. The sac is made up of
positive patients suggesting a role not only of thickened hyalinized intima. The adventitia
+ +
CD8 but also of CD4 T cells for killing virus- covering the sac is continuous with that of the
th
infected cells. parent artery. – Robbins 7 /1367
Ref:http://www.medscape.com/viewarticle/543527
94. B
90. C
Option A - With increasing age, tendons, skin,
The amount of precipitate formed is greatly
and even blood vessels lose elasticity. This is
influenced by the relative proportions of
due to the formation of cross-links between or
antigens & antibodies. If increasing quantities
within the molecules of collagen (a fibrous
of antigens are added to the same amount of
protein) that give elasticity to these tissues.
antiserum in different tubes, precipitation will
The “cross-linking” theory of aging assumes
be found to occur most rapidly & abundantly in
that with increasing age, the number of cross-
one of the middle tubes in which the antigen &
linkages within and between collagen
antibody are present in optimal or equivalent
molecules increases, leading to crystallinity
proprtions. In the preceeding tubes in which
and rigidity. – Encyclopedia Britannica
the antibody is in excess, and in the later tubes
in which the antigen is in excess, the
Option B - According to the Free-radical theory,
precipitation will be weak or even absent.
oxidative damage initiated by reactive oxygen
species is a major contributor to the functional
Ref: Textbook of Microbiology –
th decline that is characteristic of aging. While
Ananthanarayan, Paniker 6 / 89
studies in invertebrate models indicate that
animals genetically engineered to lack specific
PATHO antioxidant enzymes (such as SOD) generally
show a shortened lifespan.
91. A http://en.wikipedia.org/wiki/Reactive_oxygen_s
pecies#Cause_of_aging
Controversial question. Both TB & Gangrene
may be the answer. I prefer TB. Even Amit Option D – From the above para, it isclear that
Ashish & MK has preferred TB over gangrene decreased SOD levels cause aging (not
(AA in an old AIIMS Paper & MK in AI09 increased SOD levels?. So option D is wrong.
Paper).
Option C – Somatic mutation theory of aging -
92. C This theory states that an important part of
aging is determined by what happens to our
93. B genes after we inherit them. From the time of
conception, our body's cells are continually
reproducing. Each time a cell divides, there is a
An unruptured saccular aneurysm is a thin- chance that some of the genes will be copied
walled outpouching at an arterial branch incorrectly, this is called a mutation.
point along the circle of Willis or a major vessel Additionally, exposures to toxins, radiation or
just beyond. Saccular aneurysms measure a few ultraviolet light can causes mutations in your
millimeters to 2 or 3 cm in diameter and have a body's genes. The body can correct or destroy
bright red, shiny surface and a thin, translucent most of the mutations, but not all of them.
wall. Demonstration of the site of rupture requires Eventually the mutated cells accumulate, copy
careful dissection and removal of blood in the themselves and cause problems in the body's
unfixed brain. Atheromatous plaques, functioning related to aging.
calcification, or thromobotic occlusion of the sac
may be found in the wall or lumen of the My opinion is that the Free Radical Theory is
aneurysm. Brownish discoloration of the adjacent more appropriate as damage due to free
brain and meninges is evidence of prior radicals may produce somatic mutations also.
hemorrhage. The neck of the aneurysm may be So I will go with option B.
either wide or narrow. Rupture usually occurs at
the apex of the sac with extravasation of blood The major theories of aging as a consequence of
into the subarachnoid space, the substance of random events are:
the brain, or both. The arterial wall adjacent to
the neck of the aneurysm often shows some
intimal thickening and gradual attenuation of the

24
Online support at: www.saraswatastar.weebly.com
Crazy Complete Compendium of AIIMS May 2010
© Dr Saraswata Das. All Rights Reserved.

Cross-linking Hypertension is present in about 25% of


Wear and Tear patients and hematuria in 15%.” – Sabiston
th
Free Radical 17 / 2127
Rate of Living
Somatic Mutations 106. A FAP has AD inheritance. APC gene
on chromosome 5q21.
If u have time, please go through the various MEDICINE
theories of aging. These are described in
briefly & in a lucid way at the “American 107. A
Federation for Aging Research” website.
http://www.healthandage.com/html/min/afar/co The typical aspect of the cutis laxa patient
ntent/other2_2.htm is loose, sagging skin lacking elastic recoil.
This is in marked contrast to the
95. A hyperelasticity apparent in classical
Ehlers-Danlos syndrome.
Caspases are associated with Apoptosis,
which in turn is associated with organogenesis. Ref: Connective tissue & its heritable
disorders – Peter M. Royce, Beat U.
96. C Steinmann

Though smoking is associated with all the 4 108. A


mentioned cancers, however among these its
association is least with CA Nasopharynx. 109. D
(Confirmed by DM Oncology SR)
Cardiac tamponade, embolism & CHF
th
97. B Ref: Robbins 7 /807 have been mentioned as cardiac
th
complications of HIV in Harrison’s 17 /
98. A 1173. See the following lines from
Harrison.
99. A
As a primary consequence of HIV
100. A infection, the most common clinically
significant finding is a dilated
101. D cardiomyopathy associated with
congestive heart failure (CHF), referred to
Myeloid cell markers are: as HIV-associated cardiomyopathy.
Anti-MPO
CD13 Patients present with typical findings of
CD33 CHF, namely edema and shortness of
CD14 breath
CD117
Tamponade and death have occurred in
Ref:http://www.tatamemorialcentre.com/clinicalgu association with pericardial KS,
idelines/EBM/Vol5/pathology_final/Cytometry.htm presumably owing to acute hemorrhage.

102. C Nonbacterial thrombotic endocarditis has


been reported and should be considered
103. A in patients with unexplained embolic
phenomena.
104. C
110. C
Although all the 4 genes are associated with
CA Colon, however the lowest association is Calcium gluconate is given not to
with beta catenin. decrease the potassium level, but to
(Confirmed by DM Oncology SR) stabilize the heart in acute hyperkalemia.

105. D Insulin+glucose is the fastest agent


“Most pts with Wilm’s tumor present clinically causing intracellular shift of potassium.
with a palpable abdominal mass. Often, the pt
has no symptoms & the parents discover the 111. C
mass during bathing or the pediatrician finds it
during a routine physical examination.

25
Online support at: www.saraswatastar.weebly.com
Crazy Complete Compendium of AIIMS May 2010
© Dr Saraswata Das. All Rights Reserved.

Symptoms of hypocalcemia clinicopathologic features indistinguishable


from HFE-associated hemochromatosis
• Petechia which appear as one-off spots, then
later become rashes. Hereditary Hemochromatosis
• Perioral tingling and parasthesia, 'pins and
needles' sensation over the extremities of Hemochromatosis, HFE-related (type 1)
hands and feet. This is the earliest symptom
of hypocalcemia. C282Y homozygosity
• Tetany, carpopedal spasm are seen.
C282Y/H63D compound heterozygosity
• Latent tetany
o Trousseau sign of latent tetany Hemochromatosis, non-HFE-related
(eliciting carpal spasm by inflating
the blood pressure cuff and
Juvenile hemochromatosis (type 2A) (hepcidin
maintaining the cuff pressure mutation)
above systolic) Juvenile hemochromatosis (type 2B)
o Chvostek's sign (tapping of the (hemojuvelin mutations)
inferior portion of the zygoma will
produce facial spasms) Mutated transferrin receptor 2 TFR2 (type 3)
• Tendon reflexes are hyperactive
Mutated ferroportin 1 gene, SLC11A3 (type 4)
• Life threatening complications
o Laryngospasm
o Cardiac arrhythmias
About option D - The clinical expression of the
• EKG changes include:
disease is 5–10 times more frequent in men than
o Prolonged QTc
o Prolonged ST interval in women.
th
Ref: Harrison 17 / 2430

112. A 118. C

113. A Oral involvement in Wegener granulomatosis is


common, and autopsy studies of patients with
114. B the disease show this site is affected in nearly
all cases. Oral lesions include ulcerations and
Essential cryoglobulinemia is associated with gingival enlargement. The oral ulcerations,
HCV infection. which occur on the buccal mucosa or palate,
are the most common but least specific oral
115. B lesions. The characteristic gingival appearance
of Wegener granulomatosis is a
116. B pathognomonic finding termed "strawberry
gingivitis,"although it is less common than
117. A other findings.The gingivae take on a
characteristic swollen, reddened, and granular
Hereditary hemochromatosis is most often appearance. Initially, bright red-to-purple friable
caused by a mutant gene, termed HFE, which diffuse papules originate on the labial
is tightly linked to the HLA-A locus on interdental papillae. Involvement may
chromosome 6p (see "Genetic Basis," below). eventually include the lingual and palatal
Rarer forms of non-HFE hemochromatosis mucosa. Tooth and alveolar bone loss are
caused by mutation in other genes involved in common. Oral and skin manifestations may
iron metabolism have recently been described correlate with disease progression, thereby
providing prognostic value.
The HFE gene was identified as the most
common form of hemochromatosis in 1996. A Ref: http://emedicine.medscape.com/article/1081029-
homozygous G A mutation resulting in a overview
cysteine to tyrosine substitution at position 282
(C282Y) is the most common mutation.
Mutations in other genes involved in iron
metabolism are responsible for non-HFE-
associated hemochromatosis, including
juvenile hemochromatosis, which affects
persons in the second and third decade of life
(Table 351-1). Mutations in the genes encoding
hepcidin, transferrin receptor 2 (TfR2), and
hemojuvelin (Fig. 351-1) result in

26
Online support at: www.saraswatastar.weebly.com
Crazy Complete Compendium of AIIMS May 2010
© Dr Saraswata Das. All Rights Reserved.

119. C accompany the development of a right-to-left


th
shunt.” – Harrison 17 /1459
The skin of the face, normally around the
mouth, and the mucosa of the mouth and/or Auscultatory findings in Eisenmenger
throat, as well as the tongue, swell up over the syndrome
period of minutes to several hours. The S2
swelling can also occur elsewhere, typically in ASD – Widely split & fixed
the hands. The swelling can be itchy or painful. VSD – Single
There may also be slightly decreased PDA – Normally split
sensation in the affected areas due to P2 Accentuated
compression of the nerves. Urticaria (hives)
may develop simultaneously. Pulmonary Regurgitation murmur (Graham
Steell murmur) may be heard.
In severe cases, stridor of the airway occurs, th
with gasping or wheezy inspiratory breath Ref: Ghai 6 / 416 (Box: Fig.15.20); Nelson
th
sounds and decreasing oxygen levels. 18 /1936
Intubation is required in these situations to
prevent respiratory arrest and risk of death 124. B

125. B
Laryngeal edema is the most feared
complication of hereditary angioedema "Medulloblastoma, accounting for 90% of
(HAE) and can cause an immediate life- embryonal tumors, is a cerebellar tumor
threatening emergency. Case series indicate occurring predominantly in males and at a
that more than half the patients with HAE median age of 5–7 yr....The Chang staging
develop involvement in this area at some system, originally based on surgical
time during their lives. information, has been modified to incorporate
information from neuroimaging to identify risk
Ref: categories."
http://emedicine.medscape.com/article/135604 Ref: Nelson's Pediatrics, 18e, Chapter-497
-overview
Medulloblastoma is the most common
120. A malignant brain tumor in children, accounting
for 10-20% of primary CNS neoplasms and
121. C approximately 40% of all posterior fossa
tumors. It is a highly invasive embryonal
122. D neuroepithelial tumor that arises in the
cerebellum and has a tendency to disseminate
Surgical repair of diseased valves by incising throughout the CNS early in its course.
the fused mitral valve commissures and
replacement with prosthetic devices has For medulloblastoma, a modified version of the
greatly improved the outlook for patients with Chang staging system is commonly used. The
RHD. original Chang system was devised in the late
th th
Ref: Robbins 7 /594; Ghai 6 / 382 1960's, before the widespread use of radiology
Also see the “Addendum” on P.37. scans, and so it relies primarily on information
about tumor size and spread that is obtained
PEDIATRICS during actual surgery, with the naked eye.
Based on what the surgeon sees at the time of
123. A the surgery, the actual tumor is placed in one
of the following categories (T referring to
“The physical findings are altered when an tumor):
increase in the pulmonary vascular resistance
results in diminution of the left-to-right shunt. T1 Tumor <3 cm in diameter
Both the pulmonary outflow and tricuspid T2 Tumor >3 cm in diameter
inflow murmurs decrease in intensity, the T3a Tumor >3 cm in diameter with
pulmonic component of the second heart sound spread
and a systolic ejection sound are accentuated, T3b Tumor >3 cm in diameter with
the two components of the second heart sound definite spread into the brain stem
may fuse, and a diastolic murmur of pulmonic (part of brain that controls breathing,
regurgitation appears. Cyanosis and clubbing hearing, seeing, and other important
functions)

27
Online support at: www.saraswatastar.weebly.com
Crazy Complete Compendium of AIIMS May 2010
© Dr Saraswata Das. All Rights Reserved.

T4 Tumor >3 cm in diameter with and has a tendency to metastasize via CSF
extension up past the aqueduct of pathways.
Sylvius and/or down past the foramen
magnum Medulloblastomas are notorious for spreading
from the cerebellum down to the spine, or
In addition to "T" staging, medulloblastoma "metastasizing". They typically invade the
staging has been modified by including "M" surrounding lining tissues (meninges) before
staging, where the "M" stands for metastasis. gaining access to the cerebrospinal fluid (CSF)
Remember, this is a word that describes how which bathes both the brain and the spine.
far the tumor cells have spread from the Once there, tumor cells can travel through the
original location, if at all. The M stage is CSF and deposit themselves, or "seed", in any
determined not only by the surgeon's part of the spine, resulting in "metastatic
observations, but also in combination with MRI disease". Rarely, these tumor cells can gain
scans and lumbar cytology, and consists of 5 access outside of the CNS and metastasize to
possible groups: distant bone or bone marrow.
Ref:
http://es.oncolink.org/types/article.cfm?c=22&s
M0 No evidence of metastasis =78&ss=783&id=9484&p=2
M1 Tumor cells found in cerebrospinal
fluid (by lumbar puncture and cytology 127. A
study)
M2 Tumor beyond primary site but still 128. A
in brain
M3 Tumor deposits ("seeds") in spine 129. B
area that are easily seen on MRI
M4 Tumor spread to areas outside the 130. C
CNS (outside both brain and spine)
131. B
Each patient is assigned a combination of one
T stage and one M stage. As mentioned in the 132. B
introduction, one of the reasons staging is
important is that it helps predict how a patient 133. C
might do in the long run, or how "curable" their
cancer is, in a way. For medulloblastomas, the Compensated versus decompensated
M stage is considered far more important in shock
determining ultimate patient outcome and
survival than the T stage. In other words,
regardless of what the T stage may be, To begin to categorize and prioritize the
children who are in the M0 group do far better management of a child in shock, first determine
than those in M1, who tend to fare better than the central blood pressure. Blood pressure
M2 kids, who in turn do better than M3 or M4 measurements determine the central driving
children. Ref: pressure responsible for perfusing the most
http://es.oncolink.org/types/article.cfm?c=22&s critical organs, namely the brain and the heart.
=78&ss=783&id=9484&p=2
Minimum blood pressure requirements can be
determined by establishing the fifth percentile
for normal systolic blood pressure in a healthy,
126. A
well-perfused child. The American Heart
“Although Glioblastoma multiforme is the Association, in the course on pediatric
tumor most prone to extraneural metastasis in advanced life support (PALS), defines infants
the adult population, medulloblastoma with fifth–percentile systolic blood pressure as
metastasizes more commonly in children.” – follows:
• Newborn - 60 mm Hg
Tumors of the pediatric central nervous system
By Robert F. Keating, James T. Goodrich, • Infant (1 mo to 1 y) - 70 mm Hg
Roger J. Packer, P.369 • Child (>1 y) - 70 + 2 X age (in y)

Also know:
Thus, children with poor perfusion and blood
Medulloblastoma (WHO grade IV) is a
pressure below the parameters listed above
malignant, invasive embryonal tumor of the
cerebellum that occurs primarily in children, may be said to have decompensated shock.
has a predominantly neuronal differentiation, Such children, if not quickly and

28
Online support at: www.saraswatastar.weebly.com
Crazy Complete Compendium of AIIMS May 2010
© Dr Saraswata Das. All Rights Reserved.

aggressively resuscitated, experience carcinomas of the pancreas, breast, lung,


additional organ damage and may progress ovary, and uterus. The well-documented and
to irreversible shock and death. Children characteristic tumors include sex cord tumors
with an adequate systolic blood pressure of the ovary, adenoma malignum of the uterine
may still be in shock but may be in a state of cervix, and Sertoli cell tumors of the testis.
compensated shock. Thus, although central When gastrointestinal adenocarcinoma occurs,
perfusion to the brain and heart are still it arises from concomitant adenomatous
considered adequate, other vital organ lesions. The underlying genetic basis for
Peutz-Jeghers syndrome is the mutation of the
systems may be hypoperfused and may
gene STK11 (LKB1) located on chromosome
sustain damage that, if not reversed, 19.
progresses to decompensated shock.
Therefore, in order to determine if a patient 138. B
is in shock, many different indicators of
tissue organ perfusion must be examined. 139. B

Heart rate 140. B

141. B
Because CO depends on both SV and HR,
the body typically tries to maintain CO when 142. A (Mattox maneuver)
SV decreases by increasing the HR. Unless
the HR cannot increase for some reason 143. C
(eg, pharmacologic blockade; neurologic
damage, such as cervical cord injury; 144. B
operative insults that may be sustained
during open-heart surgery), a patient in the Complications of ileostomy – Stoma
early stages of shock is typically tachycardic. necrosis may occur in the early
However, such a sign is certainly not very postoperative period and is usually caused
sensitive in children because children may by skeletonizing the distal small bowel
be tachycardic from a wide variety of stimuli, and/or creating an overly tight fascial
including fever, pain, and agitation. defect. Limited mucosal necrosis above
Nevertheless, with the exceptions mentioned the fascia may be treated expectantly, but
above, tachycardia is generally a fairly early necrosis below the level of the fascia
and specific finding in both compensated requires surgical revision. – Schwartz
th
8 /1073
and decompensated shock.
145. A
Here the infant is 6 months old & having
SBP 85 mm Hg, which is above 70 mm Hg. 146. D
So it is a compensated shock. Also, the
infant is not tachypneic (so not early). Hence 147. A
it is a case of late decompensated
hypovolemic shock. 148. D

149. A
SURGERY
150. A
134.C
ORTHO
135. B
151. A
136. A
In 1980, the so-called Ahlbäck classification
137. D was described as follows (Ahlbäck and
Rydberg 1980).
The distribution of polyps in patients is reported
as follows: stomach, 25%; colon, 30%; and Grade 1: narrowing of the articular space;
small bowel, 100%. While these Grade 2: obliteration or almost obliteration of
hamartomatous polyps themselves do not have the articular space;
malignant potential, patients with the syndrome Grade 3: bone attrition less than 5 mm;
have an increased risk of developing Grade 4: bone attrition between 5 and 15 mm,

29
Online support at: www.saraswatastar.weebly.com
Crazy Complete Compendium of AIIMS May 2010
© Dr Saraswata Das. All Rights Reserved.

Grade 5: bone attrition greater than 15 mm. advanced age of onset of symptoms rendered
AS an unlikely diagnosis.
This grading is based on radiographic
appearance of the joint. Why not Lumbar Canal Stenosis?

From this grading, it is clear that Grade 2 Neurological signs would have been present.
means no bone attrition has yet occurred. Patients with significant lumbar spinal canal
Hence I think the best mode of treatment narrowing report pain, weakness, numbness
here is conservative. Open for discussion. in the legs while walking, or a combination
thereof.
152. A

153. A

154. B

155. C

Ankylosing spinal hyperostosis refers to


stiffness or fusion of the spine due to excessive
bone formation. It is also known as diffuse
idiopathis skeletal hyperostosis (DISH) or
ankylosing vertebral hyperostosis, and is often
incorrectly called Forestier's disease. A form of
degenerative arthritis, this disorder involves
sections of the ligaments in the spine turning
into bone--a process also known as "flowing
calcification," because it usually extends along
the sides of the vertebrae.

Ankylosing spinal hyperostosis affects three or


more vertebrae between the chest and pelvis.
We mainly see it in men, ages 50-60, along
with diabetes, heart disease, obesity and high
blood pressure. Often, physicians confuse the
stiffness in the spine that accompanies
ankylosing spinal hyperostosis with ankylosing
spondylitis.

Radiographs of the spine typically demonstrate


thoracic spinal involvement; however, diffuse
idiopathic skeletal hyperostosis (DISH) can
also affect the lumbar and cervical spine. DISH
is distinguished by the presence of flowing
syndesmophytes along, but separated from,
the anterior aspect of the vertebral bodies,
involving at least 4 levels. The disease begins
as fine ossification, 1- to 2-mm thick, but
ossification may thicken to as much as 20 mm
as the disease progresses. Radiograph of the lumbosacral spine
(anteroposterior view) showing
Why not Ankylosing spondylitis? flowing osteophytes and soft-tissue
ligamentous ossification consistent
Ankylosing spondylitis (AS) often begins in with diffuse idiopathic skeletal
young men with inflammation and tenderness
of the sacroiliac joints. However, over time, the hyperostosis.
joints will fuse and become asymptomatic and
nontender. The lack of sacroiliac tenderness in
this elderly gentleman with longstanding 156. D
symptoms, the normal sedimentation rate, and

30
Online support at: www.saraswatastar.weebly.com
Crazy Complete Compendium of AIIMS May 2010
© Dr Saraswata Das. All Rights Reserved.

CALCIUM PYROPHOSPHATE CRYSTAL Pre-ovarian cancer back pain is a common


DEPOSITION DISEASE (PSEUDOGOUT) symptom of the disease which is many-a-time
treated as a normal back pain mistakenly in the
Calcium pyrophosphate crystal deposition initial stage of cancer.
disease (CPPD), also known as pseudogout and
chondrocalcinosis, is one of the more common
disorders associated with intra-articular crystal 159. A
formation. It usually occurs in individuals over
Ovarian endometrioma (chocolate cyst) fulfills
age 50 and becomes more common with
all the criteria given in the question. Ovary is
increasing age, rising to a prevalence of 30% to
the most common site of endometriosis. It
60% in those age 85 or older. The sexes and
causes infertility & is also associated with
races are equally affected. CPPD is divided into
raised CA-125 levels.
sporadic (idiopathic), hereditary, and secondary
Normal level of CA-125: Less than 35 U/ml
types. In the hereditary variant, the crystals
A number of benign conditions can cause
develop relatively early in life and are associated
elevations of the CA 125 level, including
with severe osteoarthritis. The autosomal
pregnancy, endometriosis, uterine fibroids
dominant form of the disease has been shown to
(benign tumors), pancreatitis, normal
be related to a mutation in the ANKH gene, which
menstruation, pelvic inflammatory disease, and
encodes a transmembrane inorganic
[59] liver disease. Benign tumors or cysts of the
pyrophosphate transport channel. The
ovaries can also cause an abnormal test result.
secondary form is associated with various
Increases can also be seen in cancers other
disorders, including previous joint damage,
than ovarian cancer, including malignancies of
hyperparathyroidism, hemochromatosis,
the uterine tubes, endometrium, lung, breast,
hypomagnesemia, hypothyroidism, ochronosis,
pancreas, and gastrointestinal tract
and diabetes. The conditions leading to crystal
formation are not entirely known but include
Why not Borderline ovarian tumor?
altered activity of the matrix enzymes that
Cancer antigen 125 (CA-125) levels are not
produce and degrade pyrophosphate, resulting in
shown to aid in the diagnosis or follow-up care
its accumulation and eventual crystallization with
of patients with borderline tumors.
calcium.
160. B
CPPD is frequently asymptomatic; however, it is
a great simulator because it produces acute, 161. C
subacute, or chronic arthritis that may mimic
other disorders, such as osteoarthritis or RA. The 162. A
joint involvement may last from several days to
weeks and may be monarticular or polyarticular; Controversial question. A,C,D appear to be
the knees, followed by the wrists, elbows, true. A appears to be the best possible choice
shoulders, and ankles, are most commonly to me.
affected. Ultimately, approximately 50% of
patients experience significant joint damage. 163. C
Therapy is supportive; no known treatment
prevents or retards crystal formation. Intrahepatic cholestasis of pregnancy

th Epidemiology
Ref: Robbins 7 /1314
Incidence: 2-10% of pregnancies
Symptoms
157. A
Severe Pruritus in third trimester of
pregnancy
GYNAE & OBS Pruritus localized to trunk and extremities
Signs
No rash
158. B This ques is open for discussion. No Jaundice in mild form (Prurigo
Causes of coccydynia (coccyx pain) include gravidarum)
History of cancer - Especially colon, prostate, Labs
ovarian, cervical, testicular, or other intrapelvic Serum bile acids >4 (>16 confers adverse
malignancies. fetal outcome)
Ref:http://emedicine.medscape.com/article/309 Management
486-diagnosis Oral Antihistamines
Ursodeoxycholic Acid (Ursodiol)
Increased antepartum observation

31
Online support at: www.saraswatastar.weebly.com
Crazy Complete Compendium of AIIMS May 2010
© Dr Saraswata Das. All Rights Reserved.

Plan delivery by 38 weeks gestation Loss of pubic hair


Course
Resolves with delivery Loss of armpit hair
Complications Dry skin
Preterm delivery
Meconium-stained amniotic fluid Atrophy of vaginal mucosa
Intrauterine Fetal Demise
Return phase of tendon reflexes slow
164. C Spasm of infundibular arteries
Pituitary thrombosis
The normal pituitary gland increases in volume
during pregnancy, primarily as a result of Pituitary necrosis
estrogen-induced lactotroph hyperplasia. For
this reason, the pituitary gland is susceptible to Thyroid atrophy secondary
ischemic damage and necrosis in patients who Adrenal atrophy secondary
experience severe postpartum hemorrhage.
This entity, Sheehan's syndrome, is a rare Ovarian atrophy secondary
occurrence in developed countries today, but
still occurs occasionally. It is probably much Low basal metabolic rate
more common in developing countries, where it High cholesterol
could be a major cause of hypopituitarism.
Patients can present with acute hypopituitarism Low protein-bound iodide
in the postpartum period, or the diagnosis can
be made decades after the event in patients Low urine corticosteroid level
who have had chronic, nonspecific symptoms. Low urine estrogen level
Classically, the presenting symptom is failure
to lactate, but patients can present with a Low urine pituitary gonadotropin level
spectrum of clinical findings that reflect variable
loss of anterior pituitary gland function. Overt
diabetes insipidus is unusual in Sheehan's 165. B
syndrome, although subtle defects in posterior
pituitary function are common. The treatment 166. A
of Sheehan's syndrome includes standard
replacement therapy for hormone deficiencies. 167. C
The long-term prognosis for patients who
develop Sheehan's syndrome is permanent 168. C
hypopituitarism, requiring lifelong therapy,
although there are rare case reports of “Concurrent chemoradiation includes
recovery of pituitary function. radiation & weekly cisplatin therapy. This
therapy is found helpful to patients following
Ref: radical hysterectomy. This is also
http://journals.lww.com/theendocrinologist/Abst recommended as a primary treatment for
ract/2004/01000/Sheehan_s_Syndrome.5.aspx patients with bulky (> 4cm) tumor, stage IB or
IIA or advanced stage disease.” – Dutta
th
Gynaecology 4 /327
The list of signs and symptoms mentioned
in various sources for Sheehan Syndrome 169. C
includes the 24 symptoms listed below:
Hazards of Gestational Diabetes Mellitus
Acute shock
Increased perinatal loss is associated with
Lactation failure fasting hyperglycemia. Fetal anomalies are
however not increased. This is due to the
Asthenia absence of metabolic disturbances during
organogenesis. (GDM usually presents late in
Hypoglycemic crisis nd rd
2 or during the 3 trimester).
Absence of menstruation Macrosomia
Polyhydramnios
Menstrual irregularity Birth trauma
Recurrence of GDM in subsequent pregnancy
Pallor
is about 50%.

32
Online support at: www.saraswatastar.weebly.com
Crazy Complete Compendium of AIIMS May 2010
© Dr Saraswata Das. All Rights Reserved.

181. B
Potential candidates for GDM are those
having a previous birth of an oversized 182. B
baby of 4 kg or more.
The endolymph is produced from perilymph as
th
Ref: Dutta Obstetrics 6 / 284-285 a result of selective ion transport through the
epithelial cells of Reissner’s membrane and not
directly from the blood. The secretory tissue
170. C Mtx can’t be used during pregnancy. called the stria vascularis, in the lateral wall of
the cochlear duct, is thought to play an
171. D important role in maintaining the high ratio of
potassium ions to sodium ions in the
172. B endolymph.
173. A Ref:
http://www.britannica.com/EBchecked/topic/56
9006/stria-vascularis
OPHTHAL
“It has generally been accepted that
174. A endolymph of the cochlea is produced by stria
vascularis.” – Dr. T. Balasubramanian MS,
175. D DLO
http://www.drtbalu.co.in/endolymph.html
176. B

177. B

178. B

179. A

ENT

180. B

The most common anatomic sites of


cerebrospinal fluid (CSF) leaks are the areas of
congenital weakness of the anterior cranial fossa
and areas related to the type of surgery
performed. According to data from 53 patients
with different causes of CSF rhinorrhea, 39% of
leaks occurred in the region of the cribriform plate
and air cells of the ethmoid sinus; in 15% of
leaks, the fistula extended to the frontal sinus;
and in another 15%, the leak was in the area of
1
the sella turcica and sphenoid sinus.

Common sites of injury secondary to endoscopic


sinus surgery include the lateral lamella of the
cribriform plate and the posterior ethmoid roof
near the anterior and medial sphenoid wall.
Rarely, the leak can originate in the middle or
posterior cranial fossa and can reach the nasal
cavity by way of the middle ear and eustachian
tube.

Ref:
http://emedicine.medscape.com/article/861126-
overview
183. A (Repeat from All India 2010)

Extrinsic membranes & ligaments

33
Online support at: www.saraswatastar.weebly.com
Crazy Complete Compendium of AIIMS May 2010
© Dr Saraswata Das. All Rights Reserved.

new lesions continue to appear for 3-6 weeks.


Thyrohyoid Aching legs and swelling ankles may persist for
Thyroepiglottic weeks.
Hyoepiglottic
Cricotracheal Distribution of skin lesions - Characteristically,
lesions appear on the anterior leg; however,
Inrinsic membranes & ligaments they may appear on any surface.

Quadrangular Color of skin lesions - Lesions change color in


Cricothyroid the second week from bright red to bluish or
Cricovocal livid. As absorption progresses, the color
gradually fades to a yellowish hue, resembling a
184. A bruise. This disappears in 1 or 2 weeks as the
overlying skin desquamates.
SKIN
Joints - Arthralgia occurs in more than 50% of
185. A patients and begins during the eruptive phase or
precedes the eruption by 2-4 weeks. Erythema,
186. A swelling, and tenderness occur over the joint,
sometimes with effusions. Joint tenderness and
The explanation of this question is a bit lengthy morning stiffness may occur. Any joint may be
because I want to draw your attention to all the involved, but the ankles, knees, and wrist are
important points about these two diseases. affected most commonly. Synovitis resolves
These will not only help you to understand why within a few weeks, but joint pain and stiffness
Erythema nodosum is the answer here, but also may last up to 6 months. No destructive joint
to be prepared for any question on this topic in changes occur. Synovial fluid is acellular, and the
future. rheumatoid factor is negative

Erythema Nodosum

Erythema nodosum (EN) is an acute, nodular,


erythematous eruption that usually is limited to
the extensor aspects of the lower legs.

Age-Erythema nodosum may occur in children


and in patients older than 70 years, but it is more
common in young adults aged 18-34 years. Age
distribution varies with geographic location and
etiology.

History
The eruptive phase of erythema nodosum begins
with flulike symptoms of fever and generalized
aching. Arthralgia may occur and precedes the
eruption or appears during the eruptive phase. Classic presentation of erythema
Most lesions in infection-induced erythema nodosum with nodular red swellings
nodosum heal within 7 weeks, but active disease over the shins
may last up to 18 weeks. In contrast, 30% of
idiopathic erythema nodosum cases may last Ref:http://emedicine.medscape.com/article/10816
more than 6 months. Febrile illness with 33-overview
dermatologic findings includes abrupt onset of
illness with initial fever, followed by a painful rash Weber Christian Disease
within 1-2 days.
Weber-Christian disease is a skin condition that
Primary skin lesions - Lesions begin as red features recurring inflammation in the fat layer of
tender nodules (see the image below). Lesion the skin. The involved areas of skin manifest as
borders are poorly defined, and lesions vary from recurrent crops of erythematous, sometimes
2-6 cm. During the first week, lesions become tender, edematous subcutaneous nodules. The
tense, hard, and painful; during the second week, lesions are symmetric in distribution, and the
they may become fluctuant, as in an abscess, but thighs and lower legs are affected most
do not suppurate or ulcerate. Individual lesions frequently. Malaise, fever, and arthralgias
last approximately 2 weeks, but occasionally, frequently occur. Nausea, vomiting, abdominal

34
Online support at: www.saraswatastar.weebly.com
Crazy Complete Compendium of AIIMS May 2010
© Dr Saraswata Das. All Rights Reserved.

th
pain, weight loss, and hepatomegaly may also Robbins 7 /1265 says:
occur. Because its etiology is unknown, Weber- “Panniculitis often involves the lower legs.
Christian disease is often referred to as idiopathic Erythema nodosum presents as poorly defined,
lobular panniculitis. exquisitely tender, erythematous plaques and
nodules that may be better felt than seen. Fever
Age - Weber-Christian disease may occur in and malaise may accompany the cutaneous
young children but has been reported most signs. Over the course of weeks, lesions usually
frequently in people in the fourth to seventh flatten and become bruise-like, leaving no
decades of life. residual clinical scars, while new lesions develop.
Biopsy of a deep wedge of tissue is usually
History required to establish a definitive diagnosis.
Patients with Weber-Christian disease typically
have cutaneous and, less frequently, systemic Erythema nodosum and erythema induratum are
symptoms. but two examples among the many types of
• Patients affected with Weber-Christian panniculitis. Weber-Christian disease (relapsing
disease describe crops of lesions that febrile nodular panniculitis) is a rare form of
appear and resolve during a period of lobular, nonvasculitic panniculitis seen in children
weeks to months. The lesions are often and adults. It is marked by crops of erythematous
symmetric in distribution, and the thighs plaques or nodules, predominantly on the lower
and legs are involved most commonly. extremities, created by deep-seated foci of
Individual nodules regress during the inflammation with aggregates of foamy
course of a few weeks. histiocytes admixed with lymphocytes,
• Systemic symptoms of Weber-Christian neutrophils, and giant cells.”
disease include malaise, fever, nausea,
vomiting, abdominal pain, weight loss, ANESTHESIA
bone pain, myalgia, and arthralgia.
187. B
• The etiology of Weber-Christian disease
is unknown, and patients do not report a
history of thermal, mechanical, or Halothane impairs liver blood flow and
chemical trauma. oxygenation the most, perhaps explaining
increased incidence with its use.
Physical
Physical examination reveals erythematous, Enflurane was shown to reduce hepatic
edematous, and tender subcutaneous nodules. blood flow less than halothane.
• The nodules are usually symmetric and
measure approximately 1-2 cm;
Ref: Hepatotoxicity of inhalational agents –
however, the nodules may be much
Dept of Anesthesia, University of Sydney
larger. The lesions commonly occur on http://www.anaes.med.usyd.edu.au/lecture
the thighs and lower legs but may also s/hepatotox_clt/hepatotoxicity.html
involve the arms, trunk, and face.
• The individual nodules resolve during a
couple of weeks, leaving an atrophic Ajay Yadav has mentioned in his book that
depressed scar. Enflurane depresses hepatic functions due
to decreased hepatic blood flow (Ajay
• Occasionally, the epidermis overlying nd
Yadav 2 /63). However he has never said
the nodules breaks down, and the lesion
that Enflurane causes maximum reduction
discharges a brown liquid oil (ie,
in hepatic blood flow. So answer hee is
liquefying panniculitis).
Halothane.
• In individuals with Weber-Christian
disease with visceral involvement,
hepatomegaly or splenomegaly may be Also see the following reference:
present.
Ref:http://emedicine.medscape.com/article/10084 Enflurane (Ethrane), Isoflurane
11-overview (Forane), and Desflurane (Suprane)
Mild postoperative hepatic dysfunction--
due to altered hepatic blood flow.
These anesthetics can promote formation
of acetylator liver proteins which may
cause hepatotoxicity (type II) similar to that
caused by halothane (Fluothane);
frequency < halothane (Fluothane).
Weber Christian disease

35
Online support at: www.saraswatastar.weebly.com
Crazy Complete Compendium of AIIMS May 2010
© Dr Saraswata Das. All Rights Reserved.

http://www.pharmacology2000.com/Centra periosteal reaction. Usually, the matrix of the


l/General_Anesthesia/comp_pharm5.htm lucency is smooth and relatively
homogeneous; classically, this finding is
188. A described as a ground-glass appearance.
Irregular areas of sclerosis may be present
Contraindications to IV Regional Block with or without calcification. The lucent lesion
nd
(Bier’s Block) – Ajay Yadav 2 /114 has a thick sclerotic border and is called the
rind sign. The lesion may be surrounded by a
1. Sickle cell disease layer of thick, sclerotic reactive bone termed a
2. Raynaud’s disease rind. The classic rind is most commonly seen
3. Scleroderma in proximal femur.

Why IV Regional Block is http://emedicine.medscape.com/article/389714


contraindicated in sickle cell disease? -imaging

The presence of sickle cell disease is a PSYCHIATRY


relative contraindication to tourniquet use.
The use of a tourniquet has been 196. C
discouraged in patients who carry the
sickle cell gene, because tourniquet use 197. A
may lead to circulatory stasis, acidosis,
and hypoxemia - the triad of clinical 198. A
conditions that is known to induce sickling.
However, recent studies have suggested 199. B
that the use of a tourniquet in sickle cell
patients may not be associated with Most common substance of abuse in India
harmful effects, provided that optimum is Cannabis (not Tobacco as given in AA).
acid - base status and oxygenation are (Confirmed by Psychiatry faculty of
maintained throughout anesthesia. Before Dr.Bhatia Coaching Institute).
using a tourniquet on patients with sickle
cell disease or trait, it may be useful to test 200. B
for hemoglobin type and level. If the
decision is made to apply a tourniquet,
exsanguinate the limb carefully and Feedback / Suggestions /
closely monitor the patient's PO2 and pH. Criticisms are welcome at
IV Regional Block requires application of
saraswatastar@yahoo.com .
tourniquet. Hence it is contraindicated in Online support at
sickle cell disease. www.saraswatastar.weebly.com
189. B

RADIO

190. B

191. B

192. A

193. A

194. A

195. C

The usual appearance of fibrous dysplasia


includes a lucent lesion in the diaphysis or All the Best!
metaphysis, with endosteal scalloping and with
or without bone expansion and the absence of

36
Online support at: www.saraswatastar.weebly.com
Crazy Complete Compendium of AIIMS May 2010
© Dr Saraswata Das. All Rights Reserved.

Addendum
122. A patient presents with acute rheumatic References which indicate valve replacement is
carditis with fever. True statement is: done (of course after medical therapy has failed
a) Increased Troponin T in acute rheumatic carditis r as follows:
b) Reduced myocardial contractility
c) Signs of inflammation & necrosis
d) Valve replacement will ameliorate Heart Failure May Require Specific Treatment
CCF
Heart failure in rheumatic fever generally
Explanation: responds to bed rest and corticosteroids;
diuretics and then digoxin may be necessary in
This question has been framed on a single para [1]
in Braunwald. Read it below & you can easily rule patients with more severe disease. Digoxin
out options a,b & c. should be used with caution because of its
lowered therapeutic index in active myocarditis
and the possibility of exacerbation of heart block.
"The severity of left ventricular dysfunction, even
Surgical therapy, e.g. valve replacement, is
in the acute setting, appears to correlate with the
extent of valvulitis rather than with any occasionally required.
myocardial injury. Rheumatic myocarditis, in the
setting of preserved LV function, is not http://www.medscape.com/viewarticle/406404_5
associated with the troponin level elevation seen
[11]
in viral myocarditides. Both echocardiographic
data and postmortem pathology findings are Surgical Care
consistent with severe heart failure in acute RF,
being secondary to altered myocardial mechanics When heart failure persists or worsens after
caused by MR rather than secondary to aggressive medical therapy for acute rheumatic
myocarditis. Traditionally, the diagnosis has been heart disease, surgery to decrease valve
made on the basis of auscultation of mitral or,
less commonly, aortic insufficiency in the setting insufficiency may be life-saving.
of heart failure, with cardiomegaly in the most
severe cases. Severe MR is most commonly http://emedicine.medscape.com/article/891897-
associated with the worst prognosis—acute and treatment
sometimes refractory and fatal heart failure. This
subgroup is most likely to develop significant What is the prognosis (predicted outcome
chronic RHD, with an incidence as high as 90 and course) of the disease?
percent. There is a linear relationship between
the severity of MR during the first episode of RF
and subsequent RHD." - Braunwald's Heart
th
Disease 8 /Chapter 83 Flares tend to be unpredictable as far as how
long they will last and their severity. Having
carditis in the first attack is potentially a higher
risk for heart damage, however, complete healing
may follows carditis in some cases. The most
"Unless valvular regurgitation and severe severe heart damage may require heart surgery
congestive heart failure are refractory to drug for valve replacement.
therapy, valve surgery is avoided for acute RF
patients. Surgical morbidity and mortality have http://www.printo.it/pediatric-
been significant and failed repair leading to valve rheumatology/information/Australia/9.htm
replacement frequent, although postoperative
ventricular function generally improves
significantly, consistent with regurgitation rather There r also many case reports available which
than myocardial dysfunction being the primary show promising results with valve replacement.
mechanism leading to heart failure.” -
th
Braunwald's Heart Disease 8 /Chapter 83

37
Online support at: www.saraswatastar.weebly.com

You might also like